You are on page 1of 99

Matemáticas Financieras

Dra. Ma. Mercedes Gregorio Domínguez


Departamento de Actuaría y Seguros, ITAM

Agosto 2006
2
Índice general

I Tasas de interés 5

1. Conceptos básicos 7
1.1. Introducción . . . . . . . . . . . . . . . . . . . . . . . . . . . . . . . . . . . . . . . 7
1.2. Definición de interés . . . . . . . . . . . . . . . . . . . . . . . . . . . . . . . . . . 7
1.3. Interés y descuento parcial . . . . . . . . . . . . . . . . . . . . . . . . . . . . . . . 10
1.4. Funciones de acumulación y descuento . . . . . . . . . . . . . . . . . . . . . . . . 10
1.5. Interés y descuento efectivo . . . . . . . . . . . . . . . . . . . . . . . . . . . . . . 14

2. Tasas de interés 19
2.1. Introducción . . . . . . . . . . . . . . . . . . . . . . . . . . . . . . . . . . . . . . . 19
2.2. Capitalización de los intereses . . . . . . . . . . . . . . . . . . . . . . . . . . . . . 19
2.3. Régimen de interés simple . . . . . . . . . . . . . . . . . . . . . . . . . . . . . . . 20
2.3.1. Interés simple . . . . . . . . . . . . . . . . . . . . . . . . . . . . . . . . . . 20
2.3.2. Descuento matemático simple . . . . . . . . . . . . . . . . . . . . . . . . . 21
2.3.3. Descuento simple (descuento bancario simple) . . . . . . . . . . . . . . . . 22
2.3.4. Interés ordinario vs. interés exacto . . . . . . . . . . . . . . . . . . . . . . 23
2.4. Régimen de interés compuesto . . . . . . . . . . . . . . . . . . . . . . . . . . . . . 25
2.4.1. Tasa de interés compuesto . . . . . . . . . . . . . . . . . . . . . . . . . . . 25
2.4.2. Relación entre interés simple e interés compuesto . . . . . . . . . . . . . . 26
2.4.3. Tasa de descuento compuesto . . . . . . . . . . . . . . . . . . . . . . . . . 28
2.4.4. Relación entre descuento simple y descuento compuesto . . . . . . . . . . 28
2.4.5. Tasa nominal de interés . . . . . . . . . . . . . . . . . . . . . . . . . . . . 29
2.4.6. Tasas equivalentes . . . . . . . . . . . . . . . . . . . . . . . . . . . . . . . 30
2.4.7. Tasas de descuento nominales . . . . . . . . . . . . . . . . . . . . . . . . . 32
2.4.8. Fuerza de interés . . . . . . . . . . . . . . . . . . . . . . . . . . . . . . . . 33
2.5. Función de fuerza de interés . . . . . . . . . . . . . . . . . . . . . . . . . . . . . . 34
2.6. Tasas de interés variables . . . . . . . . . . . . . . . . . . . . . . . . . . . . . . . 37
2.7. Tasas equivalentes con diferentes regímenes de inversión . . . . . . . . . . . . . . 38
2.8. Ejemplos . . . . . . . . . . . . . . . . . . . . . . . . . . . . . . . . . . . . . . . . . 39

3. Aplicaciones de las tasas de interés 41


3.1. Problemas Típicos . . . . . . . . . . . . . . . . . . . . . . . . . . . . . . . . . . . 41
3.1.1. Monto . . . . . . . . . . . . . . . . . . . . . . . . . . . . . . . . . . . . . . 41
3.1.2. Tiempo . . . . . . . . . . . . . . . . . . . . . . . . . . . . . . . . . . . . . 42
3.1.3. Tasa de interés o descuento . . . . . . . . . . . . . . . . . . . . . . . . . . 42

3
4

3.1.4. Capital . . . . . . . . . . . . . . . . . . . . . . . . . . . . . . . . . . . . . 43
3.2. Ecuaciones de valor . . . . . . . . . . . . . . . . . . . . . . . . . . . . . . . . . . . 43
3.2.1. Fecha equivalente . . . . . . . . . . . . . . . . . . . . . . . . . . . . . . . . 47
3.2.2. Tasa de interés desconocida . . . . . . . . . . . . . . . . . . . . . . . . . . 48

II Anualidades 51

4. Anualidades 53
4.1. Concepto de anualidad . . . . . . . . . . . . . . . . . . . . . . . . . . . . . . . . . 53
4.2. Anualidades ordinarias con pagos anuales . . . . . . . . . . . . . . . . . . . . . . 55
4.2.1. Valor presente de anualidades ordinarias con pagos anuales . . . . . . . . 56
4.2.2. Monto de anualidades ordinarias con pagos anuales . . . . . . . . . . . . . 59
4.2.3. Relaciones importantes . . . . . . . . . . . . . . . . . . . . . . . . . . . . . 60
4.3. Anualidades fraccionadas . . . . . . . . . . . . . . . . . . . . . . . . . . . . . . . 60
4.3.1. Valor presente de anualidades ordinarias con pagos fracccionados . . . . . 62
4.3.2. Monto de anualidades ordinarias con pagos fraccionados . . . . . . . . . . 65
4.4. Anualidades continuas . . . . . . . . . . . . . . . . . . . . . . . . . . . . . . . . . 65
4.4.1. Valor presente de anualidades ordinarias continuas . . . . . . . . . . . . . 66
4.4.2. Monto de anualidades ordinarias continua . . . . . . . . . . . . . . . . . . 67
4.4.3. Fuerza de interés variable . . . . . . . . . . . . . . . . . . . . . . . . . . . 68
4.5. Anualidades variables . . . . . . . . . . . . . . . . . . . . . . . . . . . . . . . . . 68
4.5.1. Anualidades con variación anual y pagos anuales . . . . . . . . . . . . . . 69
4.5.2. Anualidades con variación anual y pagos fraccionados . . . . . . . . . . . 75
4.5.3. Anualidades con variación fraccionaria y pagos fraccionados . . . . . . . . 80
4.5.4. Anualidades variables continuas . . . . . . . . . . . . . . . . . . . . . . . . 83
4.6. Aplicaciones . . . . . . . . . . . . . . . . . . . . . . . . . . . . . . . . . . . . . . . 84

III Amortizaciones 87

5. Amortización 89
5.1. Concepto de amortización . . . . . . . . . . . . . . . . . . . . . . . . . . . . . . . 89
5.2. Amortización con pagos iguales . . . . . . . . . . . . . . . . . . . . . . . . . . . . 90
5.3. Amortización con un pago irregular . . . . . . . . . . . . . . . . . . . . . . . . . . 92
5.4. Fondos de amortización . . . . . . . . . . . . . . . . . . . . . . . . . . . . . . . . 93
5.5. Fondo de amortización con pagos variables . . . . . . . . . . . . . . . . . . . . . . 95
5.6. Amortizaciones con cambio de tasas . . . . . . . . . . . . . . . . . . . . . . . . . 97
5.6.1. Amortizaciones con cambio de tasas conocido de antemano . . . . . . . . 97
5.6.2. Amortizaciones cuando el cambio de tasas no se conoce de antemano . . . 98
Parte I

Tasas de interés

5
Capítulo 1

Conceptos básicos

1.1. Introducción
Las Matemáticas Financieras tienen por objeto estudiar las variaciones del valor de los bienes
económicos bajo un aspecto monetario y como función del tiempo.
Para cumplir con el objetivo se valen de la introducción de modelos matemáticos en el estudio
de las operaciones financieras en las que se invierten capitales y, pasado un tiempo, se obtiene
una rentabilidad (en forma de intereses o utilidades).
Una de las variables más importante en estos modelos será el tiempo. El tiempo puede ser
medido en distintas unidades, e.g. dias, meses, años, décadas, etc. Al hablar del tiempo podemos
referirnos a periodos de tiempo (tiempo entre dos puntos) o puntos en el tiempo (instantes), unos
u otros se ven reflejados en una línea de tiempo como se muestra en la figura 1.1.
La unidad de medición de tiempo más común es el año.

1.2. Definición de interés


Cualquier actividad financiera puede verse como la recepción de un alquiler por el uso de un
activo prestado como se muestra en la figura 1.2; la diferencia radica en las condiciones de la
transacción y en si éstas son fijas o variables, algunos ejemplos son:

Bien inmueble: Se renta un departamento (activo) por un tiempo determinado a cambio


de un alquiler fijo en unas fechas determinadas.

Préstamo: Alquiler de cierta cantidad de dinero por un tiempo determinado a cambio de


un interés que puede ser fijo o variable.

Bono: Préstamo de cierta cantidad de dinero por tiempo determinado a cambio de un


interés fijo pagado por adelantado.

Acciones: Préstamo de cierta cantidad de dinero por tiempo indeterminado a cambio de


un rendimiento incierto.

Todas las actividades financieras descansan en la costumbre de pagar un alquiler (rédito o


renta) por el uso de un activo prestado. La mayor parte de los ingresos de los inversionistas

7
8 Capítulo 1. Conceptos básicos

Periodos de tiempo

Periodo 1 Periodo 2 Periodo 3 Periodo 4


z }| {z }| {z }| {z }| {

- t

Tiempo 0 Tiempo 1 Tiempo 2 Tiempo 3 Tiempo 4

Puntos de tiempo (un instante de tiempo)

Figura 1.1: Línea de tiempo

Capital

¿Cuándo?
?
Alquiler
Deudor - Acreedor
¿Cuándo? ¿Cuánto?

6
Capital

Figura 1.2: Condiciones de una transacción financiera

se derivan del cobro de los intereses sobre préstamos o del retorno de utilidades por capitales
invertidos.
El interés puede definirse como el alquiler o rédito pagado por una persona u organización
(al que llamaremos deudor) por el uso de un activo, denominado capital, perteneciente a otra
persona u organización (al que llamaremos acreedor). Las condiciones particulares de la transac-
ción serán acordadas entre las dos partes: momento(s) en el(los) que debe pagarse el alquiler,
monto del alquiler y momento(s) en el(los) que se debe regresar el capital.
Cuando el capital está denominado en unidades monetarias recibe el nombre de principal y
el alquiler recibe el nombre de interés.
La suma de principal e interés que se debe por el uso del principal en el periodo (0,t] al
tiempo t se denomina monto al tiempo t de la operación financiera. A lo largo de este libro
consideraremos que tanto el principal como el interés están denominados en unidades monetarias
de una misma moneda.
Matemáticas Financieras. Mercedes Gregorio 9

C = M (0) M (n)

- t

0 1 2 ··· n−1 n

Figura 1.3: Monto al tiempo n

Determinante del interés es la tasa de interés. La tasa (o tipo) de interés se define como
la suma a pagar al final del periodo por unidad monetaria prestada por unidad de tiempo
convencionalmente estipulada por concepto de interés; es expresada generalmente en términos de
porcentaje. Así un préstamo convenido a la tasa de interés anual de r % significa que por cada
100 unidades de capital se pagarán r unidades de interés cada año. A la tasa de interés también se
le conoce como precio del dinero y siempre debe estar asociada a un periodo de tiempo; durante
ese periodo de tiempo el capital no varía por lo que la variación en el monto de la operación se
debe exclusivamente al efecto de los intereses. Si el interés se paga por adelantado, al principio
del periodo de la operación, el interés se denomina descuento y su determinante será la tasa
de descuento; la cual se aplicará al monto de la operación al final del periodo y no al principal.
Cuando se trata de dinero invertido en un negocio, el inversionista espera recuperar una suma
mayor que la invertida después de cierto tiempo; de esta operación surge el concepto de tasa
de retorno o tasa de rendimiento. La tasa de retorno puede definirse como la suma a recibir
por unidad de dinero invertida por unidad de tiempo convencionalmente estipulada en exceso
del monto invertido inicialmente.
En adelante sólo manejaremos el concepto de tasa de interés; pero teniendo en cuenta que
el concepto de tasa de retorno es equivalente al concepto de tasa de interés en el caso de que
tratemos con inversiones.
Sea C el principal (o capital invertido) y M (n) la cantidad recibida al final de n periodos
resultado de la inversión (esa cantidad incluye tanto capital como intereses generados hasta ese
momento). M (n) recibe el nombre de monto al tiempo n, en particular M (0) = C (Ver figura
1.3.
La diferencia entre el principal y el monto al tiempo n corresponde a los intereses generados
en el periodo de tiempo (0,n] pagados al tiempo n, denotados por I(0, n), tiempo durante el cual
estuvo invertido el capital:

M (n) − C = I(0, n) pagado al tiempo n.

Si los intereses se pagan al tiempo 0, esto es, si se pagan por adelantado, la diferencia entre
capital y monto al tiempo n recibe el nombre de descuento, D(0, n), en lugar de interés:

M (n) − C = D(0, n) pagado al tiempo 0.


10 Capítulo 1. Conceptos básicos

La diferencia en su cálculo radica en los datos que se tienen disponibles:

El interés se calcula en base al principal, I(0, n) = f (C); el interés se define en base a lo


actualmente prestado, e.g. por un préstamo de $80, se van a cobrar intereses sobre $80
pesos, se aplica una función, f (·), a esos $80 y se obtiene que se van a cobrar $20 por
interés, por lo tanto al vencimiento se van a cobrar $20+$80=$100.

El descuento se calcula en base al monto al tiempo n, D(0, n) = g(M (n)); el descuento


se define en base a lo que se tiene que regresar al vencimiento, e.g. por un préstamo por
el cual al vencimiento se van a regresar $100, se aplica una función, g(·), a esos $100 y se
obtiene que se van a cobrar $20 por descuento por adelantado, por lo tanto al inicio se van
a otorgar $100-$20=$80.

1.3. Interés y descuento parcial


El interés generado en el periodo (0, t], I(0, t), puede verse como la suma de los intereses ge-
nerados en subperiodos, donde estos subperiodos son mutuamente excluyentes (las intersecciones
entre subperiodos son vacias) y exhaustivos (cubren todo el periodo). Sean I(a, b) los intereses
generados desde el tiempo a hasta el tiempo b, a < b; el intervalo (0, b] puede expresarse como la
unión de 2 subintervalos mutuamente excluyentes y exhaustivos (0, a] y (a, b], entonces el interés
generado en el intervalo (0, b] puede verse como la suma de los intereses generados en los interalos
(0, a] y (a, b], esto es, I(0, b) = I(0, a) + I(a, b) lo cual implica que I(a, b) = I(0, b) − I(0, a).
Nota: para denotar los intereses generados durante el periodo (0,t] pueden usarse indistinta-
mente I(t) y I(0, t) al igual que para el descuento pueden usarse indistintamente D(t) y D(0, t).
Sabemos que I(a) = M (a) − C e I(b) = M (b) − C, por lo tanto, I(a, b) = M (b) − M (a). Esto
es, el interés generado en el periodo (a, b] es la diferencia entre el monto al tiempo b y el monto
al tiempo a. Así, si tenemos una partición del intervalo (0, t] de la forma (0, t1 ], (t1 , t2 ], . . . , (tn , t],
observamos

I(t) = I(0, t1 ) + I(t1 , t2 ) + . . . + I(tn , t).

De igual forma, sea D(a, b) el descuento generado desde el tiempo a hasta el tiempo b, a < b;
entonces el descuento generado en el intervalo (0, b] puede verse como la suma de los descuentos
generados en los intervalos (0, a] y (a, b], esto es, D(b) = D(a) + D(a, b).
Sabemos que D(a) = M (a) − C y D(b) = M (b) − C, por lo tanto, D(a, b) = M (b) − M (a).
Así, si tenemos una partición del intervalo (0, t] de la forma (0, t1 ], (t1 , t2 ], . . . , (tn , t], observamos

D(t) = D(0, t1 ) + D(t1 , t2 ) + . . . + D(tn , t).

1.4. Funciones de acumulación y descuento


Denominamos función de acumulación, a(t1 , t2 ), a una función mediante la cual un monto
M (t1 ) invertido al tiempo t1 se convierte en el monto M (t2 ) al tiempo t2 , cuando se le multiplica
por ella,

M (t1 )a(t1 , t2 ) = M (t2 ),


Matemáticas Financieras. Mercedes Gregorio 11

a(t1 , t2 )

M (t1 ) -M (t2 )

0 ··· t1 ··· t2

Figura 1.4: Función de acumulación

como se muestra en la figura 1.4


En el caso de que t1 = 0 y t2 = t, podemos denotar la función de acumulación como a(0, t) =
a(t), la cual denominaremos función de acumulación estándar, y representa una función mediante
la cual un capital inicial C = M (0) invertido al tiempo 0 se convierte en el monto M (t) al tiempo
t,

Ca(t) = M (t),

como se observa en la figura 1.5

a(t)

C - M (t)

0 1 2 ··· t

Figura 1.5: Función de acumulación estándar

Otro nombre que se le da al monto al tiempo t, M (t), es valor futuro al tiempo t de una
cantidad C.
Las funciones de acumulación deben cumplir con las siguientes propiedades:

1. a(t) está definida para t ≥ 0.


12 Capítulo 1. Conceptos básicos

2. a(0) = 1.
3. a(t) es, generalmente, una función no-decreciente (una función decreciente implicaría in-
tereses negativos).
4. Si los intereses crecen de forma continua a(t) es una función continua; si los intereses no
crecen continuamente entonces a(t) presenta discontinuidades .

Una propiedad que no es indispensable que las funciones de acumulación satisfagan pero que
es deseable que cumplan es el llamado Principio de consistencia: a(t1 , t2 ) = a(t1 , s)a(s, t2 )
para t1 ≤ s ≤ t2 .
En caso de que se cumpla el principio de consistencia se puede observar que a(t2 ) = a(t1 )a(t1 , t2 )
para t1 ≤ t2 por lo tanto a(t1 , t2 ) = a(t 2)
a(t1 ) ; en ese caso es posible, si no es de interés lo que ocurre
antes de t1 , hacer un cambio de variable t = t2 − t1 (con lo cual t1 se transforma en 0) y trabajar
con la función de acumulación estándar; con lo cual a(t1 , t2 ) = a(t2 − t1 ).
A veces es necesario determinar cuál es el capital que se debe invertir inicialmente para
obtener un monto M (n) al final de n periodos. Esto se puede hacer mediante la función de
descuento, denotada por a(t)−1 , que es el inverso de la función de acumulación,
µ ¶
1
M (t)a(t)−1 = M (t) = C.
a(t)
La función de descuento puede verse gráficamente como se muestra en la figura 1.6

a−1 (t)

C ¾ M (t)

0 1 2 ··· t

Figura 1.6: Función de descuento

Al valor que se obtiene de multiplicar la función de descuento por cierta cantidad se le llama
valor presente de dicha cantidad, i.e. si Xa(t)−1 = Y entonces Y es el valor presente de X y
reordenando X = Y a(t), X es el valor futuro de Y al tiempo t.
Utilizando las funciones de acumulación y descuento podemos expresar el interés y el des-
cuento en términos de las funciones f (·) y g(·) de la siguiente forma:
Si I(t) = M (t) − C = Ca(t) − C = C(a(t) ³ − 1),´entonces f (C) = C(a(t) − 1)³ y si D(n) ´ =
−1 1 1
M (n) − C = M (n) − M (n)a(n) = M (n) 1 − a(n) , entonces g(M (n)) = M (n) 1 − a(n) .

Ejemplo 1.1 Sea a(t) = 2t2 + t + 1.


Matemáticas Financieras. Mercedes Gregorio 13

1. Comprueba que a(t) cumple con las propiedades de una función de acumulación:

a) a(t) está definida ∀t ≥ 0


Está definida ya que para cada valor de t ≥ 0 tenemos un valor de a(t).
b) a(0) = 1

a(0) = 2(0)2 + 0 + 1 = 1

c) a(t) es creciente para t ≥ 0

−1
a0 (t) = 4t + 1 > 0 ⇔ t > entonces a(t) es creciente para t > 0
4

d) a(t) es continua
a(t) es un polinomio, suma de funciones continuas, por lo tanto es continua. Además
limt→x a(t) = a(x) por lo tanto es continua.

2. ¿Es consistente?

a(t2 ) 2(t2 )2 + t2 + 1
a(t1 , t2 ) = = 6= a(t2 − t1 )
a(t1 ) 2(t1 )2 + t1 + 1

por lo tanto, no es consistente.

3. Calcula el monto al tiempo n, el interés y el descuento derivados de esta función de acu-


mulación:

M (n) = Ca(n) = C(2n2 + n + 1)

I(n) = Ca(n) − C = C(2n2 + n + 1) − C = C(2n2 + n)

C(2n2 + n + 1)
D(n) = M (n) − M (n)a(n)−1 = C(2n2 + n + 1) − = C(2n2 + n)
2n2 + n + 1

4. Calcula los intereses generados entre n − 1 y n

I(n − 1, n) = M (n) − M (n − 1) = Ca(n) − Ca(n − 1)

= C(2n2 + n + 1) − C(2(n − 1)2 + (n − 1) + 1) = C(4n − 1).

Ejemplo 1.2 Sea a(t) = (1 + x)t .

1. Comprueba las siguientes propiedades:

a) a(t) está definida ∀t ≥ 0


Está definida ya que para cada valor de t ≥ 0 tenemos un valor de a(t).
14 Capítulo 1. Conceptos básicos

b) a(0) = 1

a(0) = (1 + x)0 = 1

c) a(t) = et ln(1+x) es creciente para t ≥ 0

a0 (t) = ln(1+x)et ln(1+x) ≥ 0 si x ≥ 0entonces a(t) es no decreciente para t > 0 si x ≥ 0

d) a(t) es continua
a(t) es una función exponencial, la cual es continua.

2. ¿Es consistente?

a(t2 ) (1 + x)t2
a(t1 , t2 ) = = = (1 + x)t2 −t1 = a(t2 − t1 )
a(t1 ) (1 + x)t1

por lo tanto, es consistente.

3. Calcula el monto al tiempo n, el interés y el descuento derivados de esta función de acu-


mulación:

M (n) = Ca(n) = C(1 + x)n

I(n) = Ca(n) − C = C(1 + x)n − C = C((1 + x)n − 1)

D(n) = M (n) − M (n)a(n)−1 = C(1 + x)n − C(1 + x)n (1 + x)−1 = C((1 + x)n − 1)

4. Calcula los intereses generados entre n − 1 y n

I(n−1, n) = M (n)−M (n−1) = Ca(n)−Ca(n−1) = C(1+x)n −C(1+x)n−1 = C(1+x)n−1 x.

1.5. Interés y descuento efectivo


Como ya se mencionó la tasa de interés (y en su caso la de descuento) están siempre referidas
a un periodo de tiempo de ahí surge el concepto de tasa de interés efectiva por periodo.
La tasa de interés efectiva para el periodo n es el interés que genera cierta inversión
durante el periodo n expresado como proporción de la inversión hecha al principio de ese periodo,
cuando los intereses se pagan una sola vez durante el periodo y el pago se realiza al final del
periodo (i.e. el capital permanece constante a lo largo del periodo).
Sea in la tasa de interés efectiva en el n-ésimo periodo (del tiempo n−1 al tiempo n), entonces
Matemáticas Financieras. Mercedes Gregorio 15

intereses generados durante el periodo n


in =
capital al inicio del periodo n (tiempo n − 1)
I(n − 1, n) M (n) − M (n − 1)
= =
M (n − 1) M (n − 1)
Ca(n) − Ca(n − 1)
=
Ca(n − 1)
a(n) − a(n − 1)
= para n ≥ 1, n entero.
a(n − 1)

De donde se puede ver que I(n − 1, n) = M (n − 1)in .


De la definición de tasa de interés efectiva por periodo se puede ver que las funciones de
acumulación tienen la propiedad de que pueden escribirse como producto de las tasas de interés
efectivas, así
n
Y
a(n) = (1 + i1 )(1 + i2 )(1 + i3 ) · · · (1 + in ) = (1 + ij ).
j=1

Se pueden definir tasas de interés efectivas entre dos instantes de tiempo, en lugar de tasas
de interés efectivas para un periodo determinado, esto con el objetivo de poder manejar periodos
de tiempo de diferente tamaño en un mismo problema. Denotaremos ith ,tk a la tasa de interés
efectiva para el periodo que empieza al tiempo th y termina en tk (en particular, in = in−1,n ) y
se define como:
intereses generados entre th y tk a(tk ) − a(th ) a(tk )
ith ,tk = = ⇒ 1 + ith ,tk = .
capital al momento th a(th ) a(th )
Y entonces la función de acumulación puede escribirse como:

a(0, t) = (1 + i0,t1 )(1 + it1 ,t2 )(1 + it2 ,t3 ) · · · (1 + itn ,t ),


donde los intervalos de tiempo (0, t1 ], (t1 , t2 ], . . . , (tn , t] no necesariamente tienen la misma
longitud pero son exhaustivos y mutuamente excluyentes.
La tasa de descuento efectiva en el periodo n es una medida de los intereses pagados al
comienzo del periodo y por lo tanto es la relación de los intereses generados durante el periodo
n y el capital que se tiene al final de ese periodo (al tiempo n).
Sea dn la tasa de descuento efectiva en el n-ésimo periodo (de n − 1 a n):

intereses generados durante el periodo n


dn =
capital al final del periodo n (tiempo n)
D(n − 1, n) M (n) − M (n − 1)
= =
M (n) M (n)
Ca(n) − Ca(n − 1)
=
Ca(n)
a(n) − a(n − 1)
= para n ≥ 1, n entero.
a(n)
16 Capítulo 1. Conceptos básicos

Por lo tanto, se tiene que D(n − 1, n) = M (n)dn .


Las funciones de acumulación tienen la propiedad de que pueden escribirse como producto
de las tasas de descuento efectivas así:
µ ¶µ ¶µ ¶ µ ¶ n
à !
1 1 1 1 Y 1
a(n) = ··· = .
1 − d1 1 − d2 1 − d3 1 − dn j=1
1 − dj

Se pueden definir tasas de descuento efectivas entre dos instantes de tiempo, en lugar de
tasas de descuento efectivas para un periodo determinado, esto con el objetivo de poder manejar
periodos de tiempo de diferente tamaño en un mismo problema. Denotaremos dth ,tk a la tasa de
descuento efectiva para el periodo que empieza en th y termina en tk (en particular, dn = dn−1,n )y
se define como:
intereses generados entre th y tk a(tk ) − a(th )
dth ,tk = = .
capital al momento tk a(tk )

Y entonces la función de acumulación puede escribirse como:


à !à !à ! à ! n+1
à !
1 1 1 1 Y 1
a(0, t) = ··· = ,
1 − d0,t1 1 − dt1 ,t2 1 − dt2 ,t3 1 − dtn ,t j=1
1 − dtj−1 ,tj

con t0 = 0 y tn+1 = t, donde los intervalos de tiempo (0, t1 ], (t1 , t2 ], . . . , (tn , t] no necesariamente
tienen la misma longitud pero son exhaustivos y mutuamente excluyentes.
Incluso se puede escribir la función de acumulación en términos de una mezcla de tasas de
interés y descuento efectivas siempre que cubran el periodo de manera exhaustiva y correspondan
a subperiodos mutuamente excluyentes, e.g.
à ! à !
1 1
a(0, t) = (1 + it1 ,t2 ) (1 + it2 ,t3 ) · · · ,
1 − d0,t1 1 − dtn ,t

1 a(tk )
debido a que 1 + ith ,tk = 1−dth ,tk = a(th ) .

Ejemplo 1.3 Sea a(t) = 2t2 + t + 1.

1. Determina in y dn

a(n) − a(n − 1) 2n2 + n + 1 − 2(n − 1)2 − (n − 1) − 1


in = =
a(n − 1) 2(n − 1)2 + (n − 1) + 1
2 2
2n + n − 2n + 4n − 2 − n + 1 4n − 1
= 2
= 2
2n − 4n + 2 + n − 1 + 1 2n − 3n + 2

a(n) − a(n − 1) 2n2 + n + 1 − 2(n − 1)2 − (n − 1) − 1


dn = =
a(n) 2(n)2 + (n) + 1
2n2 + n − 2n2 + 4n − 2 − 2 + 1 4n − 1
= 2
= 2
2n + n + 1 2n + n + 1
Matemáticas Financieras. Mercedes Gregorio 17

2. Demuestra que in+1 < in , ∀n > 0 entera


Para ello basta demostrar que in − in+1 > 0

4n − 1 4(n + 1) − 1
in − in+1 = −
2n2 − 3n + 2 2(n + 1)2 − 3(n + 1) + 2
8n2 + 4n − 7
=
(2n2 − 3n + 2)(2n2 + n + 1)

a) 8n2 + 4n − 7 ≥ 5, ∀n ≥ 1 por lo tanto 8n2 + 4n − 7 > 0, ∀n ≥ 1,


b) 2n2 − 3n + 2 = n(2n − 3) + 2 ≥ 4, ∀n ≥ 2 y 2(1)2 − 3(1) + 2 = 1 por lo tanto
2n2 − 3n + 2 > 0, ∀n ≥ 1
c) 2n2 + n + 1 ≥ 4, ∀n ≥ 1 por lo tanto 2n2 + n + 1 > 0, ∀n ≥ 1

Entonces, in −in+1 > 0 ⇒ in > in+1 por lo que cada periodo siguiente otorga menos interés
que el anterior.
3. Calcula el monto al tiempo 7 de $100 invertidos en t = 0.

M (7) = 100a(7) = 100[2(72 ) + 7 + 1] = 10600

4. Calcula las tasas de interés y descuento efectivas en los periodos (0,4] y (4,7]
a(4) − a(0) 2(42 ) + 4 + 1 − 1
i0,4 = = = 36
a(0) 1

a(7) − a(4) 2(72 ) + 7 + 1 − 2(42 ) − 4 − 1 69


i4,7 = = =
a(4) 2(42 ) + 4 + 1 37

a(4) − a(0) 2(42 ) + 4 + 1 − 1 36


d0,4 = = 2
=
a(4) 2(4 ) + 4 + 1 37

a(7) − a(4) 2(72 ) + 7 + 1 − 2(42 ) − 4 − 1 69


d4,7 = = =
a(7) 2(72 ) + 7 + 1 106
5. Calcula el monto al tiempo 7 de $100 invertidos en t = 0, utilizando las tasas encontradas
en el inciso anterior

µ ¶
69
M (7) = 100 (1 + i0,4 ) (1 + i4,7 ) = 100 (1 + 36) 1 +
37
à !à ! à !à !
1 1 1 1
= 100 = 100 36 69
1 − d0,4 1 − d4,7 1 − 37 1 − 106
à ! à !
1 1
= 100 (1 + i0,4 ) = 100 (1 + 36) 69
1 − d4,7 1 − 106
à ! à !µ ¶
1 1 69
= 100 (1 + i4,7 ) = 100 1+ = 10600
1 − d0,4 1 − 36
37
37
18 Capítulo 1. Conceptos básicos

Ejemplo 1.4 Sea a(t) = (1 + x)t con x ≥ 0.


1. Determina in y dn

a(n) − a(n − 1) (1 + x)n − (1 + x)n−1


in = = =x
a(n − 1) (1 + x)n−1

a(n) − a(n − 1) (1 + x)n − (1 + x)n−1 x


dn = = n
=
a(n) (1 + x) 1+x

2. ¿Es in+1 < in , ∀n > 0 entera ?


in es constante ∀n al igual que dn
3. Calcula el monto al tiempo 7 de $100 invertidos en t = 0, si x = 0.10.

M (7) = 100a(7) = 100(1 + 0.10)7 = 194.87.

4. Calcula las tasas de interés y descuento efectivas en los periodos (0,4] y (4,7]

a(4) − a(0) (1 + 0.10)4 − 1


i0,4 = = = (1.1)4 − 1 = 0.4641
a(0) 1

a(7) − a(4) (1 + 0.10)7 − (1 + 0.10)4


i4,7 = = = (1.1)3 − 1 = 0.3310
a(4) (1 + 0.10)4

a(4) − a(0) (1 + 0.10)4 − 1


d0,4 = = = 1 − (1.1)−4 = 0.3170
a(4) (1 + 0.10)4

a(7) − a(4) (1 + 0.10)7 − (1 + 0.10)4


d4,7 = = = 1 − (1.1)−3 = 0.2487
a(7) (1 + 0.10)7

5. Calcula el monto al tiempo 7 de $100 invertidos en t = 0, utilizando las tasas encontradas


en el inciso anterior

M (7) = 100 (1 + i0,4 ) (1 + i4,7 ) = 100(1.1)4 (1.1)3 = 100(1.1)7


à !à ! µ ¶µ ¶
1 1 1 1
= 100 = 100 = 100(1.1)7
1 − d0,4 1 − d4,7 1.1−4 1.1−3
à ! µ ¶
1 1
= 100 (1 + i0,4 ) = 100(1.1)4 = 100(1.1)7
1 − d4,7 1.1−3
à ! µ ¶
1 1
= 100 (1 + i4,7 ) = 100 (1.1)3 = 100(1.1)7
1 − d0,4 1.1−4
Capítulo 2

Tasas de interés

2.1. Introducción
Como ya se vio en el capítulo anterior la función de acumulación determina la tasa de interés
efectiva por periodo. En este capítulo estudiaremos algunas formas particulares de funciones de
acumulación derivadas de contratos financieros específicos.

2.2. Capitalización de los intereses


Dependiendo de las condiciones del contrato financiero puede ocurrir que al final de cier-
to periodo los intereses generados sean convertidos en capital para que a su vez generen más
intereses.
El periodo de capitalización o conversión es el intervalo de tiempo, generalmente medido
en años, al final del cual se reinvierten los intereses, i.e. los intereses son convertidos en capital.
La frecuencia de capitalización o conversión es el número de veces por año en que
el interés es sumado al capital. El periodo de capitalización es el inverso de la frecuencia de
capitalización.
De acuerdo a la forma en que es manejada la reinversión de los intereses vamos a estudiar
dos tipos de regímenes financieros:

Régimen de interés simple

Régimen de interés compuesto

En el régimen de interés simple solamente el capital genera intereses, i.e. los intereses nun-
ca son convertidos en capital; mientras en el régimen de interés compuesto los intereses son
reinvertidos cada cierto tiempo (periodo de capitalización) para generar nuevos intereses.

Ejemplo 2.1 Se invierten $500.00 al 10 % anual durante 3 años.


En el régimen de interés simple los intereses se retiran al final de cada año; en el régimen de
interés compuesto al final de cada año el capital se ve incrementado por los intereses generados
durante ese año.

19
20 Capítulo 2. Tasas de interés

Régimen de interés simple Régimen de interés compuesto


Año Capital Intereses Año Capital Intereses
1 500 50 1 500 50
2 500 50 2 550 55
3 500 50 3 605 60.50
150 165.50

M (3) = C + I(3) M (3) = C + I(3)


= 500+150 = 500+165.50
= 650 = 665.50

2.3. Régimen de interés simple


El régimen financiero de interés simple se caracteriza por la propiedad de que los intereses
obtenidos por una inversión son retirados al final de cada periodo, no se agregan al capital. Por
lo que los intereses son directamente proporcionales al capital invertido al inicio y a la duración
de la operación. El capital se mantiene constante.

2.3.1. Interés simple


Sean iS la tasa de interés simple anual y t el tiempo que se invierte un capital C (t dado en
años) entonces

I(t) = CiS t.

De donde se sigue que I(s, t) = I(t) − I(s) = CiS t − CiS s = C(iS )(t − s).
Periodo Capital Intereses en el periodo Monto
p I(p − 1, p) M (p)
1 C I(0, 1) = CiS M (1) = C + CiS = C(1 + iS )
2 C I(1, 2) = CiS M (2) = C(1 + iS ) + CiS = C(1 + 2iS )
.. .. .. ..
. . . .
t C I(t − 1, t) = CiS M (t) = C(1 + (t − 1)iS ) + CiS = C(1 + tiS )
I(t) = I(0, 1) + I(1, 2) + · · · + I(t − 1, t) = CiS t
Se observa que M (t) = C(1 + iS t), que es de la forma M (t) = Ca(t), por lo tanto, cuando
consideramos interés simple la función de acumulación es a(t) = 1 + iS t.
Podemos verificar las propiedades de la función de acumulación:

1. a(t) está definida para t ≥ 0.

2. a(0) = 1 + iS (0) = 1.

3. a0 (t) = iS ≥ 0; a(t) es una función no decreciente.

4. a(t) es una función continua.


a(t2 ) 1+iS t2
5. a(t1 ) = 1+iS t1 6= 1 + iS (t2 − t1 ) por lo tanto no se satisface el principio de consistencia.
Matemáticas Financieras. Mercedes Gregorio 21

La característica principal del interés simple es que los intereses una vez que son pagados no
generan a su vez intereses. Para el caso del interés simple podemos observar que la tasa de interés
efectiva por periodo no es constante y además va disminuyendo en el transcurso del tiempo.

a(1) − a(0) 1 + iS − [1]


i1 = = = iS
a(0) 1
a(2) − a(1) 1 + 2iS − [1 + iS ] iS
i2 = = =
a(1) 1 + iS 1 + iS
.. .. .. ..
. . . .
a(n) − a(n − 1) 1 + niS − [1 + (n − 1)iS ] iS
in = = = .
a(n − 1) 1 + (n − 1)iS 1 + (n − 1)iS

iS iS i2S
in − in+1 = − = > 0, ∀n ≥ 1 ⇒ in > in+1 .
1 + (n − 1)iS 1 + niS (1 + (n − 1)iS )(1 + niS )
Y se puede derivar su función de acumulación como:

n
Y n µ
Y ¶
iS
a(n) = (1 + ij ) = 1+
j=1 j=1
1 + (j − 1)iS
µ ¶µ ¶ µ ¶
iS iS iS
= (1 + iS ) 1 + 1+ ··· 1 +
1 + iS 1 + 2iS 1 + (n − 1)iS
µ ¶µ ¶ µ ¶
1 + 2iS 1 + 3iS 1 + niS
= (1 + iS ) ··· = 1 + niS .
1 + iS 1 + 2iS 1 + (n − 1)iS

2.3.2. Descuento matemático simple


Para t = 1 observamos que utilizando la función de acumulación de interés simple las tasas
de interés y descuento efectivas en el primer periodo están definidas por:
a(1) − a(0) 1 + iS (1) − 1
i1 = = = iS
a(0) 1

a(1) − a(0) 1 + iS (1) − 1 iS


d1 = = =
a(1) 1 + iS (1) 1 + iS
Entonces observamos que i1 = iS la tasa de interés simple por unidad de tiempo. Si definimos
dM = d1 la tasa de descuento simple por unidad de tiempo, obtenemos una relación entre las
iS dM
tasas de interés y descuento simple: dM = 1+i S
e iS = 1−d M
.
Podemos obtener la función de acumulación en términos de la tasa de descuento matemático
simple:
dM 1 − dM + dM t 1 + (t − 1)dM 1 − dM
a(t) = 1 + iS t = 1 + t= = y a(t)−1 = .
1 − dM 1 − dM 1 − dM 1 + (t − 1)dM
El descuento se calcula como:
D(t) = M (t) − C = M (t) − M (t)a(t)−1 = M (t)(1 − a(t)−1 )
µ ¶ µ ¶
1 − dM 1 + (t − 1)dM − 1 + dM M (t)dM t
= M (t) 1 − = M (t) = .
1 + (t − 1)dM 1 + (t − 1)dM 1 + (t − 1)dM
22 Capítulo 2. Tasas de interés

2.3.3. Descuento simple (descuento bancario simple)


En la práctica comercial es común simplificar el cálculo del descuento simple de tal forma
que sea calculado como D(t) = M (t)dS t, donde dS es la tasa de descuento simple por unidad
de tiempo. Lo cual implica que D(t) = M (t)(1 − a(t)−1 ) = M (t)dS t dando como resultado que
1
a(t)−1 = 1 − dS t y a(t) = 1−d St
.
En este caso para t = 1 observamos que utilizando esta función de acumulación las tasas de
interés y descuento efectivas en el primer periodo están definidas por:
1
a(1) − a(0) 1−dS (1) − 1
d1 = = 1 = dS
a(1) 1−dS (1)

1
a(1) − a(0) 1−dS (1) −1 dS
i1 = = =
a(0) 1 1 − dS

Entonces observamos que d1 = dS la tasa de descuento simple por unidad de tiempo. Si


definimos iB = i1 la tasa de interés simple por unidad de tiempo correspondiente al descuento
bancario simple , obtenemos una relación entre las tasas de interés y descuento simple bancario:
iB dS
dS = 1+i B
e iB = 1−d S
.
Podemos obtener la función de acumulación en términos de la tasa de interés bancario simple:

1 1 1 + iB 1 − (t − 1)iB
a(t) = = i = y a(t)−1 = .
1 − dS t 1 − 1+iB t
B 1 − (t − 1)iB 1 + iB

El interés se calcula como:


µ ¶ µ ¶
1 + iB 1 + iB CiB t
I(t) = M (t) − C = C −C =C −1 =
1 − (t − 1)iB 1 − (t − 1)iB 1 − (t − 1)iB

Cuadro resumen: Régimen de interés simple


F. acumulación F. descuento Interés Descuento
a(t) a(t) −1 I(t) D(t)
1
Interés simple 1 + iS t 1+iS t Ci St
1+(t−1)dM 1−dM M (t)dM t
Desc. matemático simple 1−dM 1+(t−1)dM 1+(t−1)dM
1+iB 1−(t−1)iB CiB t
Interés bancario simple 1−(t−1)iB 1+iB 1−(t−1)iB
1
Descuento simple (bancario) 1 − dS t 1−dS t M (t)dS t

Ejemplo 2.2 1. Se invierten $100 al tiempo 0 a una tasa de interés simple del 10 % anual,
¿cuál es el monto que se tiene 5 años después?

M (5) = Ca(5) = 100(1 + 0.10(5)) = 150

2. ¿Cuál es la tasa de descuento matemático simple anual equivalente a esa tasa de interés?

iS 0.10
dM = = = 0.0909 una tasa de descuento matemático simple de 9.09 % anual
1 + iS 1 + 0.10
Matemáticas Financieras. Mercedes Gregorio 23

De tal forma que

1 − dS 1 − 0.0909 0.9091
M (5)a(5)−1 = 150 = 150 = 150 = 100
1 + (t − 1)dS 1 + (5 − 1)0.0909 1.3636

3. ¿Cuáles son los intereses pagados?

M (5)dM t
D(t) = I(t) = CiS t = = 100(0.10)(5)
1 + (t − 1)dM

150(0.0909)(5)
= = 50
1 + (5 − 1)0.0909

4. ¿Cuál fue el capital invertido al tiempo 0 a una tasa de descuento simple de 9.09 % si al
final de 5 años se tienen $150?

C = M (5)a(5)−1 = 150(1 − dS (5)) = 150(1 − 0.0909(5)) = 81.825

5. ¿Cuáles serían los intereses pagados si se aplica una tasa de descuento simple de dS =
9.09 % a un monto al tiempo 5 de $150?

1 + iB iB CiB t
D(t) = I(t) = M (5)dS t = C t=
1 − iB (t − 1) 1 + iB 1 − (t − 1)iB

81.825(0.10)(5)
= 150(0.0909)(5) = = 68.175
1 − (5 − 1)(0.10)

dS 0.0909
iB = = = 0.10
1 − dS 1 − 0.0909

2.3.4. Interés ordinario vs. interés exacto


Generalmente, el régimen financiero de interés simple se aplica solamente para las operaciones
financieras de corta duración (operaciones menores a un año).
Cuando el tiempo se expresa en dias es necesario saber cómo expresarlo en términos anuales,
para ello es preciso distinguir entre el año comercial de 360 dias y el año solar de 365 dias (366
dias en años bisiestos). De esta distinción vienen derivados los conceptos de regla del banquero,
interés ordinario e interés exacto.
El interés exacto. Considera el año solar. Entre el tiempo expresado en años, t, y el tiempo
expresado en dias, g, existe la relación t = 365 óg 366 .
Por lo tanto, si la tasa de interés está expresada en años los intereses se calcularán mediante
la fórmula I(t) = CiS 365 óg 366 .
El interés mediante la regla del banquero. Considera el año comercial (año de 360 dias);
g
entre el tiempo expresado en años, t, y el tiempo expresado en dias, g, existe la relación t = 360 .
24 Capítulo 2. Tasas de interés

Por lo tanto, si la tasa de interés está expresada en años los intereses se calcularán mediante
g
la fórmula I(t) = CiS 360 .
Para los dos casos anteriores, ¿cómo calcular g? g es el número exacto de días en un periodo
determinado incluyendo el primer día o el último pero no ambos, e.g. del 2 de junio al 5 de junio,
g = 3 (2,3,4 ó 3,4,5).
El interés ordinario. Considera el año comercial; el tiempo expresado en dias considera
meses de 30 dias, para calcular el número de días se puede utilizar la fórmula: gO = 360(Y2 −
Y1 ) + 30(M2 − M1 ) + (D2 − D1 ), donde Yi representa el año de la fecha i, Mi el mes de la fecha i
y Di el día de la fecha i. Y entre el tiempo expresado en años, t, y el tiempo expresado en dias,
gO
g, existe la relación t = 360 .
Por lo tanto, si la tasa de interés está expresada en años los intereses se calcularán mediante
gO
la fórmula I(t) = CiS 360 .
El método más usado es el interés ordinario. De hecho, en este libro solamente utilizaremos
interés ordinario a menos que se indique lo contrario.
Si el periodo está denominado en meses entonces no hay diferencia en el método empleado y
m
la relación con t, el tiempo expresado en meses, m, está dado por: t = 12 .
Ejemplo 2.3 Una persona invierte $1000.00 el 1 de octubre de 2002 a la tasa de interés simple
anual del 15 %. Determinar el monto al 17 de febrero de 2003. (Usar interés ordinario, exacto y
regla del banquero)

M (t) = C(i + it) = 1000(1 + 0.15t), con t medido en años porque la tasa es anual

g = 31 + 30 + 31 + 31 + 16 = 139 , o bien
= 30 + 30 + 31 + 31 + 17 = 139
Interés mediante regla del banquero:
g 139
t= =
360 360
µ ¶ µ µ ¶¶
139 139
M = 1000 1 + 0.15 = 1057.92
360 360
Interés exacto:
g 139
t= =
365 365
µ ¶ µ µ ¶¶
139 139
M = 1000 1 + 0.15 = 1057.12
365 365
Interés ordinario:
gO = 360(2003 − 2002) + 30(2 − 10) + (17 − 1) = 360 − 240 + 16 = 136

gO 136
t= =
360 360
µ ¶ µ µ ¶¶
136 136
M = 1000 1 + 0.15 = 1056.67
360 360
Matemáticas Financieras. Mercedes Gregorio 25

Ejemplo 2.4 Un capital de $35,000.00 al 3 % de interés simple anual ha producido un monto


de $35,700.00. ¿Cuánto tiempo ha estado invertido el capital?

M (t) = Ca(t) = C(1 + it)

35, 700 = 35, 000(1 + 0.03t)

35,700
35,000 −1 0.02 2
t= = = año
0.03 0.03 3
µ ¶
g 2
t= ⇒ g = 360t = 360 = 240 días
360 3

= 8 meses (utilizando interés ordinario)

2.4. Régimen de interés compuesto


El régimen de interés compuesto considera que los intereses generados son automáticamente
reinvertidos al final de cada periodo de capitalización para obtener nuevos intereses.

2.4.1. Tasa de interés compuesto


Sea i la tasa de interés por periodo bajo el régimen de interés compuesto.
Periodo Capital Intereses en el periodo Monto
p I(p − 1, p) M (p)
1 C I(0, 1) = Ci M (1) = C + Ci = C(1 + i)
2 C(1 + i) I(1, 2) = C(1 + i)i M (2) = C(1 + i) + C(1 + i)i = C(1 + i)2
.. .. .. ..
. . . .
t C(1 + i)t−1
I(t − 1, t) = C(1 + i)t−1 i M (t) = C(1 + i)t−1 + C(1 + i)t−1 i = C(1 + i)t
I(t) = I(0, 1) + I(1, 2) + · · · + I(t − 1, t) = M (t) − M (0) = C(1 + i)t − C
Se observa que M (t) = C(1+i)t , lo que sugiere que la función de acumulación es a(t) = (1+i)t .
Podemos verificar las propiedades de la función de acumulación:

1. a(t) está definida para t ≥ 0.

2. a(0) = (1 + i)0 = 1.

3. a0 (t) = ln(1 + i)[(1 + i)t ] ≥ 0; a(t) es una función no decreciente.

4. a(t) = et ln(1+i) es una función continua.


a(t2 ) (1+i)t2
5. a(t1 ) = (1+i)t1
= (1 + i)t2 −t1 por lo tanto satisface el principio de consistencia.

Una vez que hemos definido la función de acumulación para el régimen financiero de interés
compuesto se puede observar que la tasa de interés efectiva por periodo es constante en todos los
periodos (in = i ∀n ) si y sólo si nos encontramos en el régimen financiero de interés compuesto.
26 Capítulo 2. Tasas de interés

Demostración 2.1 Por inducción:

1. ⇒

Para n = 1 :
a(1) − a(0) a(1)
i1 = i = = −1 ⇒ a(1) = (1 + i)a(0) = 1 + i.
a(0) a(0)

Suponer válido para n − 1 y probar para n. Suponer a(n − 1) = (1 + i)n−1 . Para n:

a(n) − a(n − 1) a(n)


in = i = = −1
a(n − 1) a(n − 1)

⇒ a(n) = (1 + i)a(n − 1) = (1 + i)(1 + i)n−1 = (1 + i)n .

2. ⇐ Por otro lado, si consideramos a(t) = (1 + i)t entonces la tasa de interés efectiva por
periodo para el periodo n

a(n) − a(n − 1) a(n) (1 + i)n


in = = −1= −1=i
a(n − 1) a(n − 1) (1 + i)n−1

no depende de n y por lo tanto en constate para todo periodo n.

Y la función de acumulación, como ya sabemos, resulta ser:


n
Y n
Y
a(n) = (1 + ij ) = (1 + i) = (1 + i)n .
j=1 j=1

2.4.2. Relación entre interés simple e interés compuesto


Si consideramos el valor futuro de una unidad monetaria como función del tiempo utilizando
interés simple e interés compuesto podemos observar en la figura 2.1 la relación que existe entre
ambos.
Después de observar la gráfica es claro el porqué en periodos cortos de tiempo se utiliza el
régimen de interés simple; pero, en periodos largos de tiempo se utiliza el de interés compuesto.
Como consecuencia de ello surge en la práctica la idea de pagar interés compuesto por periodos
enteros de tiempo y pagar interés simple por periodos fraccionarios.
Si el periodo fraccionario es el último, como generalmente es el caso, se puede demostrar que
el cálculo del monto total es equivalente a efectuar una interpolación entre (1 + i)n y (1 + i)n+1 ,
donde n es un número entero no-negativo (precisamente el número de periodos enteros).
Sea k, 0 < k < 1 la fracción de año que constituye el periodo final:

(1 + i)n+k ≈ (1 − k)(1 + i)n + k(1 + i)n+1


= (1 + i)n (1 − k + k + ki)
= (1 + i)n (1 + ki)
= (interés compuesto por n periodos)(interés simple por la fracción k del periodo).
Matemáticas Financieras. Mercedes Gregorio 27

Figura 2.1: Interés simple vs. interés compuesto

Ejemplo 2.5 Calcular el monto de $100,000.00 al final de 2 años 3 meses a una tasa de 4 %
semestral si,
1. se asume interés simple todo el tiempo

100, 000(1 + 0.04(4.5)) = 118, 000,

2. se asume interés compuesto todo el tiempo

100, 000(1 + 0.04)4.5 = 119, 302.63,

3. se asume interés compuesto para los periodos enteros e interés simple para el último periodo
fraccional

100, 000(1 + 0.04)4 (1 + 0.04(0.5)) = 119, 325.57.


28 Capítulo 2. Tasas de interés

2.4.3. Tasa de descuento compuesto


Ya vimos que en el caso del régimen de interés compuesto la tasa de interés efectiva por
periodo es constante. Nos gustaría definir la tasa de descuento compuesto de la misma forma:
que la tasa de descuento efectiva por periodo sea constante. Sea d la tasa de descuento efectiva
por periodo (supongamos que es constante para todos los periodos):
a(1) − a(0) a(0) 1 1
d1 = =1− =1− = d ⇒ a(1) =
a(1) a(1) a(1) 1−d

1
a(2) − a(1) a(1) a(1) 1
d2 = =1− = d ⇒ a(2) = = 1−d =
a(2) a(2) 1−d 1−d (1 − d)2
³ ´n−1
1
Vamos a suponer que a(n − 1) = 1−d , entonces
³ ´n−1
1
a(n) − a(n − 1) a(n − 1) a(n − 1) 1−d 1
dn = =1− = d ⇒ a(n) = = = .
a(n) a(n) 1−d 1−d (1 − d)n
De aquí podemos observar que a(t) = (1 − d)−t y a(t)−1 = (1 − d)t , las tasas de acumulación
y descuento respectivamente.
Debemos considerar que la función de acumulación usando una tasa de descuento efectivo y
la función de acumulación usando una tasa de interés efectiva, ambas bajo el régimen de interés
compuesto deben ser iguales, por lo tanto:
1 i d
a(t) = (1 − d)−t = (1 + i)t ⇒ =1−i⇒d= yi= .
1−d 1+i 1−d
Con lo cual encontramos la relación existente entre la tasa de interés compuesto y la tasa
de descuento compuesto válida para cualquier periodo de tiempo ya que son tasas efectivas son
constantes en el tiempo.

2.4.4. Relación entre descuento simple y descuento compuesto


De la misma forma que hicimos para las tasa de interés podemos aproximar el descuento com-
puesto con descuento compuesto para los periodos enteros y descuento simple para los periodos
fraccionarios.

µ ¶n+k µ ¶n µ ¶n+1
1 1 1
≈ (1 − k) +k
1−d 1−d 1−d
µ n ¶ µ ¶
1 k
= 1−k+
1−d 1−d
µ ¶n µ ¶
1 1 − d − k + kd + k
=
1−d 1−d
µ ¶n µ ¶
1 1 + (k − 1)d
=
1−d 1−d
= (descuento compuesto por n periodos)
×(descuento matemático simple por la fracción k del periodo)
Matemáticas Financieras. Mercedes Gregorio 29

Ahora veamos cómo se comportan los intereses cuando variamos el periodo sobre el cual se
define la tasa de interés; para ello estudiemos el siguiente ejemplo.

Ejemplo 2.6 Se invierte un capital de $100.00 durante 5 años mediante el régimen de interés
simple. ¿Cuál será el monto al final de los 5 años si

1. los periodos considerados son anuales y la tasa de interés es del 12 % anual?

2. los periodos considerados son semestrales y la tasa de interés es del 6 % semestral?

3. los periodos considerados son trimestrales y la tasa de interés es del 3 % trimestral?

4. los periodos considerados son mensuales y la tasa de interés es del 1 % mensual?

1. M (5) = 100(1 + 5(0.12)) = 160 (M (5) monto al final de 5 periodos anuales)

2. M (10) = 100(1 + 10(0.06)) = 160 (M (10) monto al final de 10 periodos semestrales)

3. M (20) = 100(1 + 20(0.03)) = 160 (M (20) monto al final de 20 periodos trimestrales)

4. M (60) = 100(1 + 60(0.01)) = 160 (M (60) monto al final de 60 periodos mensuales)

Repite el ejemplo, para el régimen de interés compuesto.

1. M (5) = 100(1 + 0.12)5 = 176.23(M (5) monto al final de 5 periodos anuales)

2. M (10) = 100(1 + 0.06)10 = 179.08 (M (10) monto al final de 10 periodos semestrales)

3. M (20) = 100(1 + 0.03)20 = 180.61 monto al final de 20 periodos trimestrales)

4. M (60) = 100(1 + 0.01)60 = 181.67 (M (60) monto al final de 60 periodos mensuales)

Como se puede observar en el caso del régimen de interés simple no existe diferencia alguna
mientras en el caso del régimen de interés compuesto al disminuir el periodo de capitalización los
intereses aumentan.
Nótese que el tiempo está expresado en periodos relativos a la deinición de la tasa de interés,
e.g. tasa trimestral periodos trimestrales.

Debido a las diferencias que existen en el régimen de interés compuesto al variar el periodo
de capitalización surge la necesidad de definir una tasa de interés que especifique el periodo de
capitalización y ésta es precisamente la tasa nominal de interés.

2.4.5. Tasa nominal de interés


La tasa de interés se establece generalmente como tasa efectiva por periodo. En el ejemplo 2.6
se consideraron tasas del 12 % efectiva anual, 6 % efectiva semestral, etc. Por interés al 6 % anual
se entiende una tasa efectiva de interés del 6 % anual convertible anualmente, i.e. el periodo de
capitalización es un año. Si el periodo de capitalización no fuese un año entonces la frecuencia
de conversión se indica expresamente, e.g. interés al 4 % anual convertible semestralmente indica
que el tiempo está medido en años y que el periodo de capitalización es un semestre; y entonces,
la tasa de interés efectiva semestral es del 2 %.
30 Capítulo 2. Tasas de interés

La tasa de interés nominal, como su nombre lo indica, sólo es una tasa de interés de nombre
no sirve para calcular los intereses (salvo en el caso que el periodo de capitalización sea igual al
periodo en que está medido el tiempo, en ese caso tanto la tasa nominal como la tasa efectiva
son iguales). Para calcular los intereses es necesario tener una tasa de interés efectiva.
Como el tiempo generalmente está medido en años existe notación específica para las tasas
nominales anuales.
Se denota por i(m) la tasa de interés nominal anual convertible m veces al año, esto es, que
existen m periodos de capitalización en un año. Entonces se tiene que la tasa de interés efectiva
(m)
para cada m-ésimo de año será i m .
Por ejemplo, una tasa nominal del 8 % anual convertible trimestralmente se denota como
(4)
i(4) = 8 % y significa que tenemos una tasa de i 4 = 0.08 4 = 2 % efectiva trimestral (y por
consiguiente el tiempo debe estar medido en trimestres).
Como hemos visto, la tasa i(m) , tasa nominal anual convertible m veces al año tiene 2 impli-
caciones:

1. existen m periodos de capitalización en un año


i(m)
2. m es la tasa efectiva por m-ésimo de año.

Sabemos que si utilizamos el régimen de interés compuesto entonces la función de acumulación


es de la forma a(t) = (1 + i∗)t∗ donde i∗ es la tasa de interés efectiva por cada uno de los
t∗ periodos. Si queremos determinar la función de acumulación utilizando una tasa de interés
nominal hacemos uso de las implicaciones arriba mencionadas y obtenemos que para el tiempo
t tenemos mt periodos de m-ésimo de año y la tasa de interés efectiva en cada uno de esos mt
(m)
periodos es i m entonces:
à !mt
i(m)
a(t) = 1 + .
m

Ejemplo 2.7 ¿Cuál será el monto que producirá un capital de $1000.00 después de 5 años
invertidos a una tasa del 6 % anual convertible semestralmente?
(2)
i(2) = 0.06 ⇒ la tasa efectiva semestral es i 2 = 0.03. Hay 10 semestres en 5 años, por lo
tanto: ³ ´2(5)
M (10) = 1000 1 + 0.06
2 = 1000(1 + 0.03)10 = 1, 343.92.

Como ya se mencíonó, generalmente las tasas nominales serán anuales; pero nada impide que
se definan tasas nominales cuando la unidad de tiempo no es el año; si se tiene una tasa x nominal
por periodo convertible y veces en el periodo esto implica una tasa efectiva xy por y-ésimo de
periodo, e.g. una tasa nominal semestral convertible trimestralmente de 6 % implica una tasa de
interés del 62% = 3 % efectivo trimestral (porque existen 2 trimestres en un semestre).

2.4.6. Tasas equivalentes


Se dice que dos tasas anuales de interés con diferentes periodos de conversión son equivalentes
si producen el mismo interés compuesto al final del mismo periodo.

Ejemplo 2.8 Al final de un año el monto de $10,000 invertidos al tiempo 0 es


Matemáticas Financieras. Mercedes Gregorio 31

1. al 6 % anual convertible cuatrimestralmente


µ ¶3
0.06
10000 1 + = 10, 612.08
3

2. al 6.1208 % efectivo anual

10000(1 + 0.061208) = 10, 612.08

Por lo tanto, las tasas del 6 % anual convertible cuatrimestralmente y 6.1208 % efectivo men-
sual son equivalentes.

Para encontrar la relación que existe entre una tasa nominal anual i(m) y una tasa efectiva
anual calculamos el monto de una unidad monetaria al final de un año usando cada una de las
tasas de interés e igualamos los montos para obtener las tasas equivalentes:
à !m à !mt
i(m) i(m)
1+ =1+i⇒ 1+ = (1 + i)t
m m

de donde se obtiene que:


à !m
i(m)
i= 1+ −1
m

y también
1
i(m) = m[(1 + i) m − 1].

De forma similar para las tasas que tienen diferentes convertibilidades se observa que:
à !m à !n
i(m) i(n)
1+ = 1+
m n

lo que implica
Ã !n 
i (n) m
(m)
i = m 1 + − 1 .
n

Ejemplo 2.9 Un banco ofrece el 20 % de interés efectivo anual.

1. ¿Cuál es la tasa de interés efectiva mensual equivalente?


à !12
i(12) i(12) 1
1+ = 1 + i = 1.20 ⇒ = 1.2 12 − 1 = 0.0153
12 12

Esto es, una tasa de 1.53 % efectiva mensual.


32 Capítulo 2. Tasas de interés

2. ¿Cuál sería la tasa nominal semestral convertible mensualmente correspondiente?


Esto corresponde a la tasa efectiva mensual multiplicada por el número de meses que hay
en un semestre: 6(0.0153) = 0.0918. Esto es una tasa de 9.18 % semestral convertible
mensualmente.

3. ¿Cuál sería la tasa nominal anual convertible mensualmente correspondiente?


Esto corresponde a la tasa efectiva mensual multiplicada por el número de meses que hay
en un año: i(12) = 12(0.0153) = 0.1836. Esto es una tasa de 18.36 % anual convertible
mensualmente.

Ejemplo 2.10 Encontrar la tasa de interés nominal anual convertible trimestralmente equiva-
lente a una tasa del 18 % nominal anual convertible semestralmente.
à !2 à !4
i(12) i(4)
1+ = 1+
2 4

Ã !2  "µ #
(2) 4 ¶1
i 0.18 2
i(4) = 4  1 + − 1 = 4 1 + − 1 = 0.1761
2 2

Obteniendo i(4) = 17.61 %

2.4.7. Tasas de descuento nominales


De igual manera que para las tasas de interés se puede definir para las tasas de descuento la
tasa nominal de descuento. La tasa nominal de descuento anual convertible m veces al año, d(m) ,
es una medida de los intereses pagados al comienzo de cada m-ésima parte de año.
(m)
Si d(m) es la tasa de descuento nominal anual convertible cada m-ésimo de año, entonces d m
es la tasa de descuento efectiva para cada m-ésimo de año.
Así podemos definir la función de acumulación utilizando una tasa de descuento nominal
anual convertible m veces al año como
à !−mt
d(m)
a(t) = 1− ,
m

mientras la función de descuento será


à !mt
−1 d(m)
a(t) = 1− .
m

El capital resultante de descontar una unidad monetaria que se encuentra al final de un año
a las tasas de descuento efectiva anual, d, y de descuento nominal anual, d(m) , cuando estas tasas
son equivalentes es
à !m
d(m)
1−d= 1−
m

de donde se tiene que


Matemáticas Financieras. Mercedes Gregorio 33

à !m
d(m)
d=1− 1−
m

y también
1
d(m) = m[1 − (1 − d) m ].

De forma similar para tasas que tienen diferentes convertibilidades se observa que:
à !m à !n
d(m) d(n)
1− = 1− .
m n

Además, siguiendo la misma idea tenemos que


à !m
i(m) 1
1+ =³ ´n .
m 1− d(n)
n

2.4.8. Fuerza de interés


Hemos observado que cuanto más pequeño es el periodo de convertibilidad más intereses se
generan; pero cada vez es menor el incremento, lo que nos hace pensar que existe un límite
cuando al considerar i(m) hacemos tender m a infinito.
Consideremos la cantidad δ = ln(1 + i) donde i es la tasa de interés efectiva anual. Sabemos
que
à !m
i(m)
1+i= 1+
m

por lo tanto,
à !
i(m)
δ = ln(1 + i) = m ln 1 +
m

obteniendo
δ
i(m) = m(e m − 1).
³ ´
x2 x3
Si escribimos la exponencial como una serie de Taylor ex = 1 + x + 2! + 3! + · · · entonces
tenemos que:

 ³ ´2 ³ ´3 
δ δ
 δ m m 
i(m) = m 1 + + + + · · · − 1
m 2! 3!

δ2 δ3
= δ+ + + ···
2m 6m2
34 Capítulo 2. Tasas de interés

Tomamos el límite cuando m tiende a infinito ( o sea capitalización continua ) y obtenemos

lı́m i(m) = δ,
m→∞

donde δ es la tasa instantánea de interés o fuerza de interés cuando i es la tasa de interés


efectiva anual.
¿Qué pasa si tomamos lı́mm→∞ d(m) ? Ã !m
1 1 1
Como 1−d = 1 + i entonces − ln(1 − d) = ln(1 + i) = δ. Además 1−d = (m) por
1− d m
³ (m)
´
consiguiente δ = − ln(1 − d) = −m ln 1 − d m .
Siguiendo el mismo razonamiento que en el caso de la tasa de interés:

³ −δ
´
d(m) = m 1 − e m
 ³ ´2 ³ ´3 
−δ −δ
 −δ m m 
= m 1 − 1 − − − − · · ·
m 2! 3!
" #
δ δ2 δ3
= m − + − ···
m 2m2 6m3
δ2 δ3
= δ− + − ···
2m 6m2

Obteniendo lı́mm→∞ d(m) = δ. Con lo que observamos que la tasa de interés continuo y la
tasa de descuento continuo son iguales. Esto debido a que si la unidad de tiempo es un instante
el principio y el final del periodo coinciden en el mismo punto.
Cuadro resumen: Régimen de interés compuesto
F. acumulación F. descuento
a(t) a(t)−1
Interés compuesto efectivo (1 + i)t (1 + i)−t
Descuento compuesto efectivo (1 − d) −t (1 − d)t
³ ´mt ³ ´−mt
i(m) i(m)
Interés compuesto nominal 1+ m 1+ m
³ ´−mt ³ ´mt
d(m) (m)
Descuento compuesto nominal 1− m 1 − dm
Fuerza de interés eδt e−δt

2.5. Función de fuerza de interés


Una forma más general de entender la fuerza de interés es como tasa de crecimiento de una
población. Consideremos una población; sea f (t) el número de individuos de la población al
tiempo t y ∆ un número pequeño. Entonces,

f (t + ∆) − f (t)
f (t)

nos da la tasa de crecimiento de la población en el periodo de tiempo de tamaño ∆, y


Matemáticas Financieras. Mercedes Gregorio 35

f (t + ∆) − f (t)
∆f (t)

nos da la tasa de crecimiento por unidad de tiempo.


Para obtener la tasa de crecimiento instantáneo tomamos el límite cuando ∆ tiende a cero:

f (t + ∆) − f (t) 1 f (t + ∆) − f (t)
lı́m = lı́m (2.1)
∆→0 ∆f (t) f (t) ∆→0 ∆
1 df (t)
= (2.2)
f (t) dt
d ln f (t)
= = δ(t) (2.3)
dt
(2.4)

Donde δ(t) es la tasa de crecimiento en cada instante de tiempo.


Integrando ambos lados de (2.3) desde 0 a s tenemos:

Rs
f (s) δ(t)dt
= e 0 .
f (0)
Rs
Por lo tanto, f (s) = f (0)e 0 δ(t)dt nos da el número de individuos en la población al tiempo
s en relación al número de individuos que había al tiempo cero.
Si consideramos f (0) = C = M (0) como R el capital inicial a invertir y f (s) = M (s) el monto de
s
δ(t)dt
la inversión al tiempo s, entoncesR a(s) = e 0 se puede ver como una función de acumulación.
s
− δ(t)dt
Y en consecuencia, =e a−1 (s)
es una válida función de descuento.
0

Podemos verificar las propiedades de la función de acumulación:

1. a(t) está definida para t ≥ 0.


R0
δ(s)ds
2. a(0) = e 0 = e0 = 1.
Rt
δ(s)ds
3. a0 (t) = δ(t)e 0 ≥ 0 si δ(s) ≥ 0, ∀s ≥ 0; a(t) es una función no decreciente.

4. a(t) es una función continua.


R t2 R t2 R t2 −t1
δ(s)ds δ(s)ds
a(t2 ) eR0 δ(s)ds
5. a(t1 ) = t1 =e t1 lo cual no necesariamente es igual a e 0 por lo que
δ(s)ds
e 0
si satisface o no el principio de consistencia dependerá de la forma de δ(t).

En general, δ(t), la cual llamaremos función de fuerza de interés, puede ser cualquier función
del tiempo siempre que sea positiva para toda t. Y se puede establecer una relación uno a uno
entre funciones de acumulación y funciones de fuerza de interés. Observando que,
36 Capítulo 2. Tasas de interés

Rt
δ(s)ds
e 0 = a(t)
Z t
δ(s) = ln(a(t))
0
d
d a(t)
δ(t) = ln(a(t)) = dt .
dt a(t)

Y se pueden calcular las tasas de interés efectivas por periodo, para t1 < t2
R t2 R t1
δ(s)ds δ(s)ds R t2
a(t2 ) − a(t1 ) e 0 −e 0 δ(s)ds
it1 ,t2 = = R t1 =e t1 − 1,
a(t1 ) e 0
δ(s)ds

Rn
δ(s)ds
en particular, in = e n−1 − 1.

Ejemplo 2.11 (régimen


R de interés compuesto) Para el caso del interés compuesto sabemos que
t
a(t) = (1 + i)t ⇒ e 0 δ(s)ds = (1 + i)t ⇒ δ(t) = ln(1 + i) = δ. Esto es, δ(t) es contante ∀t; δ es la
tasa instantánea de interés o fuerza de interés cuando i es la tasa de interés efectiva anual.

Ejemplo 2.12 (Régimen R de interés simple) Para el régimen de interés simple tenemos que
t R
a(t) = 1 + iS t, entonces e 0 δ(s)ds = 1 + iS t ⇒ 0t δ(s)ds = ln(1 + iS t) ⇒ δ(t) = 1+i
iS
St
.

³ ´k
1
Ejemplo 2.13 Si δ(t) = 1+t ln B, k > 1, calcula el monto de $1 al tiempo. Además calcula
la tasa de interés efectiva por periodo para esta inversión.
(Z µ ¶k )
t 1
M (t) = Ca(t) = exp ln Bds
0 1+s

½ µ ¶¾
ln B 1
M (t) = exp 1−
k−1 (1 + t)k−1

(Z )
n µ 1
¶k
in = exp ln Bds
n−1 1+s

½ µ ¶¾
ln B 1 1
in = exp − .
k−1 nk−1 (1 + n)k−1
Matemáticas Financieras. Mercedes Gregorio 37

2.6. Tasas de interés variables


Siguiendo con la misma idea podemos incluso considerar funciones de fuerza de interés que no
sólo varían con el tiempo sino que también cambian su forma en el tiempo, e.g. para un intervalo
de tiempo (0,t]


 δ1 (s)
0 < s ≤ t1
δ(s) = δ2 (s) t1 < s ≤ t2

 δ (s) t < s ≤ t
3 2
R t1
δ1 (s)ds
R t2
Sabemos que a(t) = R(1 + i0,t1 )(1 + it1 ,t2 )(1 + it2 ,t ), además i0,t1 = e 0 − 1, it1 ,t2 =
t
δ2 (s)ds δ3 (s)ds
e t1 − 1 y it2 ,t = e t2 − 1 por lo tanto
µ Rt ¶ µ R t2 ¶µ Rt ¶ Rt
1
δ1 (s)ds δ2 (s)ds δ3 (s)ds δ(s)ds
a(t) = e 0 e t1 e t2 =e 0 .

En particular, si tenemos una fuerza de interés que es constante en ciertos intervalos de tiem-
po, i.e. consideramos el régimen de interés compuesto donde la tasa varía en distintos periodos,
tendríamos:


 δ1 0 < s ≤ t1
δ(s) = δ2 t1 < s ≤ t2

 δ 3 t2 < s ≤ t

µ Rt ¶ µ R t2 ¶µ Rt ¶
1
δ1 ds δ2 ds δ3 ds
a(t) = e 0 e t1 e t2 = eδ1 t1 eδ2 (t2 −t1 ) eδ3 (t−t2 )

Ejemplo 2.14 Considera una tasa de interés efectiva anual del 12 % durante los primeros 4
años y del 10 % durante los siguientes 4 años. Calcula la función de acumulación.
Podemos resolver el problema de 3 formas distintas:

Se puede calcular directamente


(
(1.12)t 0<t≤4
a(t) =
(1.12)4 (1.1)t−4 4 < t ≤ 8

debido a que la función de acumulación del régimen de interés compuesto satisface el prin-
cipio de consistencia.
Mediante tasas de interés efectivas
• Para el periodo [0,t), 0 ≤ t < 4
a(t) − a(0)
i0,t = = a(t) − 1 = (1 + 0.12)t − 1.
a(0)

• Para el periodo [4,t], 4 ≤ t ≤ 8


a(t) − a(4) a(t) (1 + 0.10)t
i4,t = = −1= − 1 = (1 + 0.10)t−4 − 1.
a(4) a(4) 1 + 0.104
38 Capítulo 2. Tasas de interés

(
1 + i0,t = (1.12)t 0≤t<4
a(t) =
(1 + i0,4 )(1 + i4,t )(1.12)4 (1.1)t−4 4 ≤ t ≤ 8

Mediante funciones de fuerza de interés


(
δ1 (s) = ln(1 + 0.12) = 0.1133 0 ≤ s < 4
δ(s) =
δ2 (s) = ln(1 + 0.10) = 0.0953 4 ≤ s ≤ 8

 Rt
 e 0
δ1 (s)ds
= eln(1.12)t = (1.12)t 0≤t<4
a(t) = R4 Rt
 δ1 (s)ds+ δ2 (s)ds
e 0 4 = e4 ln(1.12) e(t−4) ln(1.1) = (1.12)4 (1.1)t−4 4≤t≤8

2.7. Tasas equivalentes con diferentes regímenes de inversión


Ya vimos que en el caso de interés compuesto, dos tasas son equivalentes si producen el
mismo interés al final de un mismo periodo; y serán equivalentes independientemente del periodo
considerado.
En el caso de considerar distintos regímenes de inversión las tasas serán equivalentes exclu-
sivamente en un periodo de inversión dado. Esto es, se tiene que determinar en qué periodo se
desea calcular la equivalencia para poder determinar las tasas equivalentes.

Ejemplo 2.15 Calcula la tasa de interés simple anual equivalente a una tasa del 10 % efectivo
anual en un periodo de 1 semestre, 1 año y 5 años.

1.10.5 −1
1. 1 + iS (0.5) = (1 + 0.10)0.5 ⇒ iS = 0.5 = 0.0976
1.1−1
2. 1 + iS (1) = (1 + 0.10)1 ⇒ iS = 1 = 0.1

1.15 −1
3. 1 + iS (5) = (1 + 0.10)5 ⇒ iS = 5 = 0.1221

Ejemplo 2.16 Un operador financiero ofrece un instrumento de inversión a 5 años con una
tasa de interés simple anual del 7 %; si se quiere ofrecer otro instrumento de inversión a 5
años definido mediante una fuerza de interés anual constante que otorgue exactamente el mismo
interés, ¿cuál debe ser la fuerza de interés que debe ofrecer?

1
1 + 0.07(5) = eδ(5) ⇒ δ = ln(1 + 0.07(5)) = 0.06.
5
¿Cuál será la tasa de interés equivalente si se desea ofrecer una tasa nominal anual convertible
semestralmente?
à !10
i(2) 1
1 + 0.07(5) = 1+ ⇒ i(2) = 2{(1 + 0.07(5)) 10 − 1} = 0.0609.
2
Matemáticas Financieras. Mercedes Gregorio 39

2.8. Ejemplos
Ejemplo 2.17 1. Utilizando interés simple

a) ¿Cuál es el monto de $1,000.00 invertidos durante 8 meses al 10 % anual de interés


simple?
b) ¿En cuántos semestres se convertirán $1000.00 en $1400.00 invertidos al 10 % anual
de interés simple?
c) ¿A qué tasa de interés simple mensual se convertirán $1000.00 en $1400.00 invertidos
durante 6 años?

2. Repite las preguntas anteriores utilizando interés compuesto.

1. Interés simple
³ ´ ³ ´
8 8
a) M 12 = 1000 1 + 12 (0.10) = 1066.67
¡ ¢ ³ ´³ ´
b) 1400 = 1000 1 + 2s (0.10) ⇒ s = 1400
1000 −1 2
0.10 = 8 semestres
³ ´³ ´
1400 1
c) 1000(1 + 6(12)i) = 1400 ⇒ i = 1000 −1 6(12) = 0.0056 = 0.56 %

2. Interés compuesto
8
a) 1000(1 + 0.10) 12 = 1065.60
s
1000 )
ln( 1400
b) 1000(1 + 0.10) 2 = 1400 ⇒ s = 2 ln(1+0.10) = 7.06 semestres
h ³ ´i
1 1400
c) 1000(1 + i)6(12) = 1400 ⇒ i = exp 6(12) log 1000 − 1 = 0.0047 = 0.47 %

Ejemplo 2.18 Considera la siguiente fuerza de interés

δ(t) = 0.08 + 0.025t, 0 ≤ t ≤ 5.

1. Encuentra la función de acumulación al tiempo t

Rt Rt
δ(s)ds 0.08+0.025sds
a(t) = e 0 =e 0

t2
= e0.08t+0.025 2

2. Encuentra la tasa de interés efectiva anual equivalente en el periodo (0, m]

m2
a(t) = (1 + i)m = e0.08m+0.025 2

µ ¶1
m2 m
⇒1+i = e0.08m+0.025 2

m
⇒ i = e0.08+0.025 2 − 1

O bien,
40 Capítulo 2. Tasas de interés

m2
a(t) = eδm = e0.08m+0.025 2

m
⇒ δ = 0.08 + 0.025
2
m
⇒ i = eδ − 1 = e0.08+0.025 2 −1

3. Calcula la tasa de interés efectiva por periodo

a(n) − a(n − 1) a(n)


in = = −1
a(n − 1) a(n − 1)
n2
e0.08n+0.025 2
= (n−1)2
−1
e0.08(n−1)+0.025 2
= e0.0675+0.025n − 1

O bien,
Rn
(0.08+0.025s)ds
in = e n−1 − 1 = e0.0675+0.025n − 1.

Ejemplo 2.19 Considera la siguiente función de acumulación

a(t) = 2t2 + t + 1
encuentra la función de fuerza de interés equivalente.
Rt d
δ(s)ds dt a(t) 4t + 1
a(t) = e 0 ⇒ δ(t) = = .
a(t) 2t2 + s + 1
Ejemplo 2.20 Se tienen $100,000 que van a ser invertidos durante 3 años. El primer año la
tasa de descuento anual convertible mensualmente es de 6 %, el 2o. año la tasa de interés anual
convertible semestralmente es de 5 % y el 3er. año la fuerza de interés es de 4 % anual. Calcula
la tasa de interés efectiva anual constante para los 3 años equivalente al esquema de inversión
presentado para los 3 años.
à !12 à !2 à !12 µ ¶2
1 i(2) δ 1 0.05
a(3) = 1+ e = 1+ e0.04 = (0.995)−12 (1.025)2 e0.04 = 1.1613
1− d(12) 2 1 − 0.06
12
2
12

1
a(3) = (1 + i)3 = 1.1613 ⇒ i = (1.1613) 3 − 1 = 0.0511.

Ejemplo 2.21 Si la tasa de interés efectiva por periodo es

ik = (1 + r)k (1 + i) − 1.
Encuentra la función de acumulación en n
n
Y n(n+1)
a(n) = (1 + r)k (1 + i) = (1 + r) 2 (1 + i)n .
k=1
Capítulo 3

Aplicaciones de las tasas de interés

3.1. Problemas Típicos


Cualquier contrato financiero tiene especificadas básicamente cuatro variables asociadas con
la figura 1.2.

Capital invertido.

Tiempo de la inversión (¿cuándo se regresa el capital?).

Tasa de interés o descuento (y el correspondiente régimen de inversión) (¿cuánto es el


alquiler y cuándo se paga?) .

Monto del capital al final del periodo de inversión (¿cuándo se regresa el capital?).

Si tres de esas cantidades son conocidas entonces es posible determinar la cuarta.

3.1.1. Monto
Calcular el monto que produce un capital invertido durante cierto tiempo a cierta tasa de
interés o descuento.

Ejemplo 3.1 Determinar el monto de un capital de $100.00 para los siguientes casos:

1. Al 2.5 % de interés semestral simple durante 6 años


Tasa de interés definida en semestres por lo tanto el tiempo debe estar en semestres:

M (12) = Ca(12) = 100(1 + 0.025(12)) = 130.00

2. Al 4 % de interés anual convertible trimestralmente durante 10 años:

µ ¶40
0.04
M (40) = Ca(40) = 100 1 + = 148.89
4

41
42 Capítulo 3. Aplicaciones de las tasas de interés

3. A una tasa de descuento del 6 % bianual convertible anualmente durante 8 años:


à !8
1
M (8) = Ca(8) = 100 = 127.59
1 − 0.06
2

4. Al 5 % efectivo anual durante 10 años, 4 % efectivo anual durante los siguientes 5 años y
2.5 % efectivo anual durante los últimos 3 años:

M (18) = Ca(18) = 100(1 + 0.05)10 (1 + 0.04)5 (1 + 0.025)3 = 213.42

5. A la tasa instantánea del 4 % anual durante 5 años

M (5) = Ca(5) = 100e0.04(5) = 122.14

3.1.2. Tiempo
Calcular el tiempo que tiene que estar invertido un capital para convertirse en un monto
determinado bajo cierta tasa de interés o descuento.

Ejemplo 3.2 ¿En cuántos años se doblará el dinero a una tasa de interés efectiva anual de 6 %?

M (t) = 2C = Ca(t) = C(1 + 0.06)t ⇔ 2 = 1.06t

ln 2
⇔ ln 2 = t ln 1.06 ⇒ t = = 11.89566 = 11 años y 327 dias (aproximadamente)
ln 1.06
Si se usa interés simple para el periodo fraccionario entonces:
2
(1.06)11
−1
(1.06)11 (1 + 0.06k) = 2 ⇒ k = = 0.89292
0.06
es decir, 11 años y 326 dias (aproximadamente).

3.1.3. Tasa de interés o descuento


Calcular la tasa de interés o descuento necesaria para que cierto capital se convierta en el
monto deseado después de un periodo determinado.

Ejemplo 3.3 ¿ A qué tasa de interés anual convertible trimestralmente se tienen que invertir
$1000.00 para en 6 años obtener $1600.00?
à !24
i(4)
M (24) = 1600 = Ca(24) = 1000 1 +
4

"µ ¶ 1 #
(4) 1600 24
⇒i =4 − 1 = 0.0791
1000
Matemáticas Financieras. Mercedes Gregorio 43

3.1.4. Capital
Calcular el valor presente de cierto monto bajo una tasa de interés o descuento determinada
en cierto periodo de tiempo.

Ejemplo 3.4 Calcular el valor presente de $200.00 pagaderos al final de 10 años suponiendo
una tasa de descuento del 5 % convertible cuatrimestralmente.
µ ¶30
0.05
C = M (30)a(30)−1 = 200 1 − = 120.80
3

3.2. Ecuaciones de valor


Hasta ahora hemos considerado que el pago de capital e intereses se realiza en una misma
operación; pero, ¿qué pasa cuando el pago de estas cantidades consiste de varias operaciones en
diferentes puntos de tiempo?
Para poder analizar cantidades en diferentes puntos de tiempo es necesario especificar todas
las cantidades en el mismo punto de tiempo, al que llamaremos fecha focal o fecha de valuación;
ya que, cantidades es diferentes puntos de tiempo no son comparables.
De forma general consideremos que cada una de las cantidades de interés está formada por
un conjunto de flujos de efectivo.
Definiremos un conjunto de flujos de efectivo como dos conjuntos de datos, un conjunto
de montos {S1 , . . . , Sn } y un conjunto de fechas {t1 , . . . , tn } donde Si corresponde al pago u
obligación pagadero en el momento ti .

Ejemplo 3.5 Considera el siguiente préstamo, se darán $100,000 hoy y $140,000 en un año por
los cuales se cobrarán intereses al 4 % efectivo anual pagaderos al final de cada año durante 7
años. Al final del 5to. año se empezará a regresar el capital a $80,000 cada año. Muestra los
conjuntos de flujos de efectivo que representan cada una de las cantidades involucradas.

Capital: {(100000, 140000), (0,1)}.

Intereses: {(4000, 9600, 9600, 9600, 9600, 6400, 3200),(1,2,3,4,5,6,7)}.

Regreso del capital: {(80000, 80000, 80000),(5,6,7)}.

Una ecuación de valor es una igualdad entre varios conjuntos de flujos de efectivo valuados
en la fecha focal o fecha de valuación; i.e. cada uno de los elementos de los conjunto de flujos
debe valuarse en valor presente o futuro dependiendo donde se encuentren respecto de la fecha
focal.
Para facilitar los cálculos de una ecuación de valor es conveniente graficar unidimensional-
mente los conjuntos de flujos de efectivo por medio de un diagrama de tiempo.

Ejemplo 3.6 Considera dos conjuntos de flujos de efectivo, A y B: A = {{S1A , S2A }, {tA A
1 , t2 }} y
B B
B = {{S1 }, {t1 }}.

Si se escoge como fecha focal t, entonces


44 Capítulo 3. Aplicaciones de las tasas de interés

S2A -

S1A -

0 tA
1 tB
1 tA
2 t

S1B -

Si se escoge como fecha focal 0, entonces

¾ S2A

¾ S1A

tA
1 tB
1 tA
2 t

¾ S1B

Si se escoge como fecha focal s, entonces

¾ S2A

S1A -

tA
1 tB
1 s tA
2 t

S1B -

Las ecuaciones de valor sirven para resolver problemas en los cuales se tiene un conjunto de
flujos de efectivo y se desea formar otro conjunto de flujos de efectivo que sea equivalente; pero
que tenga condiciones específicas.
Matemáticas Financieras. Mercedes Gregorio 45

Ejemplo 3.7 Hugo tiene dos deudas una de $500.00 a pagar en un año y otra de $1000.00 a
pagar en 3 años. En un nuevo arreglo convino en pagar $750.00 hoy y el resto dentro de 4 años,
¿qué cantidad tendrá que pagar al final del cuarto año para que los arreglos sean equivalentes si
consideramos una tasa de interés simple anual del 5 %?
Sea X la cantidad que debe pagarse al final del cuarto año. Si tomamos como fecha de valua-
ción el tiempo t = 4

1000 -

500 -

0 1 2 3 4

750 -
X

500a(3)+1000a(1) = 750a(4)+Xa(0) ⇔ 500(1+0.05(3))+1000(1+0.05(1)) = 750(1+0.05(4))+X(1+0.05(0))

⇒ X = 500(1.15) + 1000(1.05) − 750(1.20) = 725

Se tendrán que pagar $725.00 al final del año 4.


De igual forma podríamos resolver el problema si escogemos otra fecha focal. Si tomamos
como fecha de valuación el tiempo t = 0

¾ 1000

¾ 500

0 1 2 3 4
750

¾ X

500 1000 750 X


500a(1)−1 +1000a(3)−1 = 750a(0)−1 +Xa(4)−1 ⇔ + = +
1 + 0.05(1) 1 + 0.05(3) 1 + 0.05(0) 1 + 0.05(4)

µ ¶
500 1000
⇒X= + − 750 (1.20) = 714.91
1.05 1.15
46 Capítulo 3. Aplicaciones de las tasas de interés

Se tendrán que pagar $714.91 al final del año 4.


Como se puede observar el resultado es distinto dependiendo de la fecha focal utilizada.
¿Qué pasa si en lugar de considerar interés simple utilizamos interés compuesto?
Si tomamos como fecha focal t = 4

500a(3)+1000a(1) = 750a(4)+Xa(0) ⇔ 500(1+0.05)3 +1000(1+0.05)1 = 750(1+0.05)4 +X(1+0.05)0

⇒ X = 500(1.05)3 + 1000(1.05) − 750(1.05)4 = 717.18


Se tendrán que pagar $717.18 al final del año 4.
Si tomamos como fecha focal t = 0
500 1000 750 X
500a(1)−1 +1000a(3)−1 = 750a(0)−1 +Xa(4)−1 ⇔ + = +
(1 + 0.05) (1 + 0.05)3 (1 + 0.05)0 (1 + 0.05)4
µ ¶
500 1000
⇒X= + − 750 (1.05)4 = 717.18
(1.05) (1.05)3
Se tendrán que pagar $717.18 al final del año 4.
En el ejemplo hemos observado como mientras para el régimen de interés simple el resultado
varía si tomamos en cuenta diferentes fechas focales, para el caso del interés compuesto no varía
no importando qué fecha focal se utilice. Esto tiene que ver con el principio de consistencia, si la
función de acumulación que se utilice satisface el principio de consistencia entonces no importa
la fecha focal que se utilice siempre se obtendrá el mismo resultado.
Ejemplo 3.8 Como reembolso de los pagos de $200.00 al final de 4 años y de $500.00 al final
de 10 años se conviene en pagar $300.00 de forma inmediata y realizar un pago adicional al final
de 3 años. Se pide calcular el monto del pago adicional si se considera una tasa de interés del
6 % anual convertible trimestralmente. Utilizar como fecha focal t = 3 (tiempo medido en años).

¾ 500

¾ 200

0 4 8 12 16 20 24 28 32 36 40

300 -
X

µ ¶−4 µ ¶−28 µ ¶12


−1 −1 0.06 0.06 0.06
200a(4) +500a(28) = 300a(12)+Xa(0) ⇔ 200 1 + +500 1 + = 300 1 + +X
4 4 4

200 500
X= 4
+ − 300(1.015)12 = 159.30
1.015 1.01528
Se deben pagar $159.30 al final del tercer año.
Matemáticas Financieras. Mercedes Gregorio 47

Hasta ahora al considerar ecuaciones de valor la incógnita siempre ha sido el valor del pago
para que la nueva serie de pagos sea igual a la anterior. Pero también puede preguntarse por
otras variables como la fecha a la cual debe hacerse un determinado pago o la tasa de interés.

Ejemplo 3.9 Se adeudan $3400.00 pagaderos al final de un año y $5600.00 al final de 2 años.
Si se hace un pago en este momento de $4500.00 ¿ en qué fecha deberá hacerse un segundo pago
de $4500.00 si la tasa de interés es del 8 % anual convertible semestralmente?

3400a(2)−1 + 5600a(4)−1 = 4500a(0) + 4500a(t)−1


µ ¶−2 µ ¶−4 Ã !
0.08 0.08 0.08 −t
⇔ 3400 1 + + 5600 1 + = 4500 + 4500 1 +
2 2 2

³ ´
3400(1.04)−2 +5600(1.04)−4 −4500
ln 4500
⇒t=− = 6.919866 semestres,
ln(1.04)

esto es, 3.46 años, o bien, 3 años 168 dias (aproximadamente).

3.2.1. Fecha equivalente


Otro problema que se puede resolver mediante ecuaciones de valor es el de encontrar la fecha
equivalente o vencimiento promedio.
La fecha equivalente es la fecha en que debe hacerse un sólo pago que es la suma de todas las
deudas para que el nuevo conjunto de flujos de efectivo sea igual al anterior. Esto es, si se tiene
un conjunto de flujos de efectivo O = {{S1 , . . . , Sn }, {t1 , . . . , tn }} se desea encontrar el tiempo t
al cual debe realizarse el pago S = S1 + . . . + Sn para que el conjunto de flujos {{S}, {t}} sea
equivalente a O.
Si se considera como fecha focal el tiempo 0, entonces:

S1 a(t1 )−1 + S2 a(t2 )−1 + . . . + Sn a(tn )−1 = (S1 + S2 + . . . + Sn )a(t)−1

de donde se obtiene que

S1 a(t1 )−1 + S2 a(t2 )−1 + . . . + Sn a(tn )−1


a(t)−1 = .
S1 + S2 + . . . + Sn

Si utilizamos el régimen de interés compuesto y definimos la variable v = (1 + i)−1 , entonces:

S1 v t1 + S2 v t2 . . . + Sn v tn
vt = ,
S1 + S2 + . . . + Sn

obteniendo
³ ´
S1 v t1 +S2 v t2 ...+Sn v tn
ln S1 +S2 +...+Sn
t= .
ln v
48 Capítulo 3. Aplicaciones de las tasas de interés

Ejemplo 3.10 Encontrar la fecha equivalente para liquidar deudas de $500.00, $1000.00 y $1500.00
pagaderos dentro de 6 meses, un año y dos años respectivamente. La tasa de interés es del 7 %
anual convertible semestralmente.
Trabajando con la tasa de .07
2 = 0.035 efectiva semestral:
³ ´
500(1.035)−1 +1000(1.035)−2 +1500(1.035)−4
ln 500+1000+1500
t= = 2.8
ln(1.035−1 )
esto es, 2.8 semestres; o bien 1.4 años.
³ ´2
.07
Si trabajamos con la tasa efectiva anual equivalente i = 1 + 2 − 1=0.071225:
³ ´
500(1.071225)−0.5 +1000(1.071225)−1 +1500(1.071225)−2
ln 500+1000+1500
t= = 1.4
ln(1.071225−1 )
esto es, 1.4 años.

3.2.2. Tasa de interés desconocida


Un problema regularmente encontrado en la práctica es el de determinar la tasa de interés
que haga dos conjuntos de flujos de efectivo equivalentes. Para poder determinar el valor de la
incógnita se utilizan principalmente 4 métodos:
Encontrar el valor de la tasa de interés directamente, despejando i de la ecuación de valor.
Esto es posible cuando los conjuntos de flujos de efectivo están formados por un sólo flujo.
Resolviendo mediante técnicas algebraicas para el valor de i. Esto puede hacerse cuando la
ecuación de valor puede verse como un polinomio y pueden encontrarse las raíces de éste.
Mediante interpolación lineal de tasas.
Mediante técnicas numéricas, e.g. Newton-Raphson, bisección, etc.
Ejemplo 3.11 Se desea conocer cual es la tasa de interés efectiva anual a la cual pagos de
$100.00, $200.00 y $300.00 en uno, dos y tres años respectivamente se convertirán al final de 3
años en $700.00.
La ecuación de valor correspondiente a este problema es:

100(1 + i)−1 + 200(1 + i)−2 + 300(1 + i)−3 = 700(1 + i)−3

⇔ 100(1 + i)2 + 200(1 + i) − 400 = 0


1. Observamos que es imposible despejar directamente de esta ecuación el valor de i.
2. Podemos resolver el problema algebraicamente, encontrando las raíces del polinomio 100x2 +
200x − 400 = 0
p (
−200 ± 2002 − 4(100)(−400) 1.2361
x= =
2(100) −3.236 1 + i no puede ser negativo

x = 1 + i = 1.2361 ⇒ i = 0.2361.
Matemáticas Financieras. Mercedes Gregorio 49

3. Mediante interpolación lineal


Dados x1 , x2 , f (x1 )yf (x2 ) el estimador de f (x0 ) con x1 ≤ x0 ≤ x2 es
µ ¶
x0 − x1
f (x0 ) = f (x1 ) + (f (x2 ) − f (x1 )) .
x2 − x1

Si se desea encontrar la raíz de una ecuación, se desea encontrar x0 tal que f (x0 ) = 0,
entonces
µ ¶
x0 − x1
0 ≈ f (x1 ) + (f (x2 ) − f (x1 ))
x2 − x1

µ ¶
x2 − x1
⇒ x0 ≈ x1 − f (x1 ) .
f (x2 ) − f (x1 )

Sea f (x) = 100x2 + 200x − 400. Consideremos x1 = 1.20 ⇒ f (x1 ) = −16 y x2 = 1.25 ⇒
f (x2 ) = 6.25, entonces
µ ¶
1.25 − 1.20
x0 ≈ 1.20 − (−16) = 1.2360 ⇒ i ≈ 0.2360.
6.25 − (−16)

4. Newton-Raphson
Pertenece a los métodos de iteración de punto fijo. El punto p es llamado punto fijo de una
función g(x) si g(p) = p. Empezando con un valor inicial p0 , la secuencia {pn }∞ n=0 puede
ser construida como p1 = g(p0 ), p2 = g(p1 ), . . . , pn+1 = g(pn ). La secuencia resultante
puede o no converger a p. Si |g 0 (p)| < 1 y p0 está lo suficientemente cerca de p convergerá.
Los métodos de iteración de punto fijo pueden ser usados para resolver la ecuación f (x) = 0,
transformando la ecuación original a la forma g(x) = x. Las soluciones a la ecuación
original, f (x) = 0, serán las mismas que las de la transformada g(x) = x. El método de
Newton-Raphson propone como función de punto fijo para encontrar la solución de f (x) = 0
a g(x) = x − ff0(x)
(x) de tal forma que la sucesión estará dada por:

f (xn )
xn+1 = xn − .
f 0 (xn )

En nuestro caso sea x0 = 1.20, como f (x) = 100x2 +200x−400 entonces f 0 (x) = 200x+200

f (x0 ) 100(1.20)2 + 200(1.20) − 400


x1 = x0 − = 1.20 − = 1.2364
f 0 (x0 ) 200(1.20) + 200
f (x1 ) 100(1.2364)2 + 200(1.2364) − 400
x2 = x1 − 0 = 1.2364 − = 1.2361
f (x1 ) 200(1.2364) + 200
f (x2 ) 100(1.2361)2 + 200(1.2361) − 400
x3 = x2 − 0 = 1.2361 − = 1.2361
f (x2 ) 200(1.2361) + 200
50 Capítulo 3. Aplicaciones de las tasas de interés
Parte II

Anualidades

51
Capítulo 4

Anualidades

4.1. Concepto de anualidad

Una anualidad es una serie de flujos de efectivo con pagos periódicos, generalmente
iguales, que se efectúan durante la existencia de una situación determinada.
Dependiendo de las características de los pagos se tienen diferentes tipos de anualidades.
La anualidad se denomina cierta si se fijan a priori el número, cuantía de los pagos y momentos
en que se realizan los pagos.
La anualidad se denomina contingente o aleatoria si el número, cuantía de los pagos y/o
momentos en que se realizan los pagos dependen de la ocurrencia o no de algún suceso.
Si la cuantía de los diferentes pagos es constante la anualidad se denomina constante. En otro
caso, si los pagos varían en su cuantía, se denominan anualidades variables. En el caso particular
de una anualidad constante donde los pagos anuales son iguales a uno (o en caso de pagos con
frecuencia distinta a la anual, la suma de la cuantía de los pagos durante el año es 1), entonces
se denomina anualidad ordinaria.
Si el número de pagos es finito se denomina anualidad temporal y perpetuidad en el caso de
un número de pagos infinito.
Con referencia a un determinado momento, la anualidad es inmediata si el primer pago se
realiza en el primer periodo; mientras será diferida m periodos si el primer pago se realiza en el
periodo m + 1.
La anualidad es anticipada si los pagos se realizan al principio de cada periodo de pago y es
vencida si se realizan al final de cada periodo de pago.
Si X representa el primer pago de la anualidad entonces gráficamente las anualidades pueden
representarse como se muestra en la figura 4.1.
El valor presente de una anualidad es la suma de los valores presentes de los pagos, valuados
al tiempo 0.
El monto de una anualidad es la suma de los montos de los pagos, valuados al final del último
periodo de pago.
Vamos a trabajar con anualidades ciertas y, en esta primera parte, constantes bajo el régimen
de interés compuesto.
Para el valor presente de anualidades ordinarias se considera la siguiente notación:

53
54 Capítulo 4. Anualidades

inmediata diferida X= primer


anticipada anticipada pago
X X

0 1 2 3 ··· m − 1 m m + 1
X X
inmediata diferida
vencida vencida

Figura 4.1: Representación gráfica de las anualidades

anticipada número de pagos en una


(no se pone si es vencida) ? unidad de tiempo
?
(m) (si m = 1 no se pone)
h|än|
diferida h unidades 6 6 duración de la anualidad
de tiempo (medida en unidades de tiempo)
(no se pone si es inmediata)

Para el monto o valor futuro de anualidades ordinarias se considera la siguiente notación:

anticipada número de pagos en una


(no se pone si es vencida) ? unidad de tiempo
?
(m) (si m = 1 no se pone)
s̈n|
6 duración de la anualidad
(medida en unidades de tiempo)

En caso de que pudiese prestarse a confusión el valor de la tasa de interés efectiva anual
a utilizar en determinada anualidad debe ponerse explícitamente del lado inferior derecho del
símbolo de valor presente o valor futuro según se requiera. Por ejemplo, si la tasa de interés
Matemáticas Financieras. Mercedes Gregorio 55

(m)
efectiva anual es del 5 % entonces el valor presente de la anualidad se denotará h| än|i=5 % .
1
Para un mejor manejo de los valores presentes de las anualidades se define, v = 1+i , el valor
presente de una unidad monetaria en un año.

4.2. Anualidades ordinarias con pagos anuales


Las anualidades ordinarias con pagos anuales son series de pagos donde la cuantía de cada
pago es $1; y se hace un pago cada año, durante n años si es una anualidad temporal o para
siempre si es una perpetuidad.
Dependiendo del momento en que se realiza el primer pago vamos a considerar las siguientes
anualidades:

Anualidad inmediata anticipada, el primer pago se realiza en el primer periodo de pago y


todos los pagos se hacen al principio de cada periodo de pago.

1 1 1 1 ···

0 1 2 3 ···

Anualidad inmediata vencida, el primer pago se realiza en el primer periodo de pago y


todos los pagos se hacen al final de cada periodo de pago.

1 1 1 ···

0 1 2 3 ···

Anualidad diferida h periodos anticipada, el primer pago se realiza en el periodo de pago


h + 1 y todos los pagos se hacen al principio de cada periodo de pago.
56 Capítulo 4. Anualidades

1 1 1 ···

0 ··· h h+1 h+2 ···

Anualidad diferida h periodos vencida, el primer pago se realiza en el periodo de pago h + 1


y todos los pagos se hacen al final de cada periodo de pago.

1 1 ···

0 ··· h h+1 h+2 ···

En el caso de anualidades temporales a n años el número total de pagos es n, en el caso de


perpetuidades el número total de pagos es infinito.

4.2.1. Valor presente de anualidades ordinarias con pagos anuales


Como ya se mencionó el valor presente de una anualidad es la suma de los valores presentes
de los flujos de efectivo.

Anualidad temporal por n años. Consiste en una serie de n pagos. Respecto al momento
en que se hace cada pago tenemos:

• Anualidad inmediata anticipada. La notación del valor presente es: än| . El valor pre-
sente es la suma de n valores presentes:

n−1
X 1 − vn 1 − vn
än| = 1 + v + v 2 + . . . + v n−1 = vt = =
t=0
1−v d
Matemáticas Financieras. Mercedes Gregorio 57

• Anualidad inmediata vencida. La notación del valor presente es: an| . El valor presente
es la suma de n valores presentes:
n
X 1 − vn 1 − vn
an| = v + v 2 + . . . + v n = vt = v =
t=1
1−v i

• Anualidad diferida h años anticipada. La notación del valor presente es: h| än| . El valor
presente es la suma de n valores presentes:
h+n−1
X n−1
X
h| än| = v h + v h+1 + v h+2 + . . . + v h+n−1 = vt = v t+h = v h än|
t=h t=0

• Anualidad diferida h años vencida. La notación del valor presente es: h| an| . El valor
presente es la suma de n valores presentes:
h+n
X n
X
h+1 h+2 h+3 h+n t
h| an| =v +v +v + ... + v = v = v t+h = v h an|
t=h+1 t=1

Perpetuidad. Consiste en una serie con un número infinito de pagos. Se puede considerar
que una perpetuidad es una anualidad temporal a n años cuando tomamos el límite de n
a infinito. Respecto al momento en que se hace cada pago tenemos:

• Perpetuidad inmediata anticipada, el primer pago se realiza en el primer periodo de


pago y todos los pagos se hacen al principio de cada periodo de pago. La notación del
valor presente es: ä∞| . El valor presente es:

1 − vn 1
ä∞| = lı́m än| = 1 + v + v 2 + . . . = lı́m =
n→∞ n→∞ d d
• Perpetuidad inmediata vencida, el primer pago se realiza en el primer periodo de
pago y todos los pagos se hacen al final de cada periodo de pago. La notación del
valor presente es: a∞| . El valor presente es:

1 − vn 1
ä∞| = lı́m än| = v + v 2 + . . . = lı́m =
n→∞ n→∞ i i
• Perpetuidad diferida h años anticipada, el primer pago se realiza en el periodo de pago
h + 1 y todos los pagos se hacen al principio de cada periodo de pago. La notación
del valor presente es: h| ä∞| . El valor presente es:
µ ¶
h h+1 h+2 h h 1
h| ä∞| = lı́m ä
n→∞ h| n|
=v +v +v + . . . = lı́m v än| = v
n→∞ d
• Perpetuidad diferida h años vencida, el primer pago se realiza en el periodo de pago
h + 1 y todos los pagos se hacen al final de cada periodo de pago. La notación del
valor presente es: h| a∞| . El valor presente es:
µ ¶
h+1 h+2 h+3 h h 1
h| a∞| = lı́m a
n→∞ h| n|
=v +v +v + . . . = lı́m v an| = v
n→∞ i
58 Capítulo 4. Anualidades

Ejemplo 4.1 El beneficiario de un seguro recibe de parte de la Cía. de seguros 2 ofertas:


1. $100,000 de contado
2. $11,000 por año, al final de cada año, durante 12 años
Diga cuál oferta es mejor si la tasa de interés vigente en el mercado es del 8 % efectiva anual.

100000

0 1 2 ··· 11 12

11000 11000 ··· 11000 11000

1. 100,000
2. 11000v + 11000v 2 + 11000v 3 + . . . + 11000v 12
= 11000[v + v 2 + v 3 + . . . + v 12 ] = 11000a12|
³ ´ ³ ´
1−v 12 1−(1.08)−12
= 11000 i = 11000 0.08 = 82, 896.86

Ejemplo 4.2 Un profesionista dona $1,500,000 a la universidad a fin de que se proporcione una
beca anual (al principio de cada año) de forma indefinida comenzando dentro de un año. Si el
dinero puede ser invertido al 7 % anual, ¿cuál es el importe de la beca?

X X X X ···

0 1 2 3 4 ···

X1| ä∞| = Xa∞|

µ ¶ Ã !
1 1 1 1
1| ä∞| =v = i = = a∞|
d 1+i 1+i
i
Matemáticas Financieras. Mercedes Gregorio 59

1500000 1500000
X= = 1 = 1500000(0.07) = 105, 000
a∞| i

4.2.2. Monto de anualidades ordinarias con pagos anuales


El monto o valor futuro de una anualidad es la suma de valores futuros de sus pagos, tomando
como fecha de valuación el final del último periodo de pago.
No hay diferencias entre el monto de anualidades diferidas y no diferidas, lo único que importa
es el número de periodos de pago. Así como tampoco existen montos de perpetuidades, ya que
no hay último periodo de pago.

Anualidad temporal n años. Consiste en una serie de n pagos anuales. Respecto al momento
en que se hace cada pago tenemos:

• Anualidad inmediata (o diferida) anticipada. La notación del monto es: s̈n| . El valor
futuro es:
n
X 1 − (1 + i)n (1 + i)n − 1
s̈n| = (1+i)n +(1+i)n−1 +. . .+(1+i) = (1+i)t = (1+i) =
t=1
1 − (1 + i) d

• Anualidad inmediata (o diferida) vencida. La notación del monto es: sn| . El valor
futuro es:
n−1
X 1 − (1 + i)n (1 + i)n − 1
sn| = (1 + i)n−1 + (1 + i)n−2 + . . . + 1 = (1 + i)t = =
t=0
1 − (1 + i) i

Ejemplo 4.3 Una compañía piensa hacer 10 depósitos de $10,000.00 cada uno al principio del
año empezando en el año 5 en un fondo que gana el 12 % anual, ¿cuánto habrá en el fondo al
final del año en el que se hace el último depósito ?

10000 10000 10000 · · · 10000 10000 10000

0 ··· 4 5 6 ··· 11 12 13

10000(1 + i)10 + 10000(1 + i)9 + . . . + 10000(1 + i)

= 10000[(1 + i)10 + (1 + i)9 + . . . + (1 + i)] = 10000s̈10|


60 Capítulo 4. Anualidades

à ! à !
(1 + i)10 − 1 (1.12)10 − 1
= 10000 = 10000 0.12 = 196, 545.83
d 1.12

4.2.3. Relaciones importantes


1 = ian| + v n

h| an| = ah+n| − ah|

än| = (1 + i)an| = 1 + an−1|

h| än| =h−1| an| = ah+n−1| − ah−1| = äh+n| − äh|

sn| = (1 + i)n an|

an| = v n sn|

1 1
an| = sn| +i

h| an| = v h+n sn|

s̈n| = sn+1| − 1

s̈n| − sn| = (1 + i)n − 1

än| − an| = 1 − v n

1 1
än| = s̈n| +d

4.3. Anualidades fraccionadas


Se define la anualidad fraccionada ordinaria de frecuencia m como la anualidad que pro-
1 1
porciona m -ésimo de unidad monetaria en cada m -ésimo de año. Por lo tanto, se trata de una
anualidad que proporciona un pago de $1 al año, distribuido uniformemente a lo largo del año.
Cada periodo de pago consiste de un m-ésimo de año. Los pagos se realizan durante n años si es
una anualidad temporal o para siempre si es una perpetuidad.
Dependiendo del momento en que se realiza el primer pago vamos a considerar las siguientes
anualidades:

Anualidad inmediata anticipada, el primer pago se realiza en el primer periodo de pago,


1
(0, m ]], y todos los pagos se hacen al principio de cada periodo de pago.
Matemáticas Financieras. Mercedes Gregorio 61

1 1 1 1 1 1 1 1 1 1
m m m ··· m m m ··· m m m ··· m

1 2 m−1 m+1 2m−1 2m+1 nm−1


0 m m ··· m 1 m ··· m 2 m ··· m n

1
Anualidad inmediata vencida, el primer pago se realiza en el primer periodo de pago, (0, m ],
y todos los pagos se hacen al final de cada periodo de pago.

1 1 1 1 1 1 1 1 1 1
m m ··· m m m ··· m m m ··· m m

1 2 m−1 m+1 2m−1 2m+1 nm−1


0 m m ··· m 1 m ··· m 2 m ··· m n

Anualidad diferida h periodos anticipada, el primer pago se realiza en el periodo de pago


1
mh + 1, (h, h + m ] y todos los pagos se hacen al principio de cada periodo de pago.

1 1 1 1 1 1 1 1
m m ··· m m m ··· m m ··· m

mh+1 m(h+1)−1 m(h+1)+1 m(h+2)+1 m(h+n)−1


0 ··· h m ··· m m ··· m ··· m
h+1 h+2 h+n

Anualidad diferida h periodos de tiempo vencida, el primer pago se realiza en el periodo


62 Capítulo 4. Anualidades
³ i
1
de pago mh + 1, h, h + m , y todos los pagos se hacen al final de cada periodo de pago.

1 1 1 1 1 1 1 1
m ··· m m m ··· m m ··· m m

mh+1 m(h+1)−1 m(h+1)+1 m(h+2)+1 m(h+n)−1


0 ··· h m ··· m m ··· m ··· m
h+1 h+2 h+n

En el caso de anualidades temporales a n años el número total de pagos es n, en el caso de


perpetuidades el número total de pagos es infinito.

4.3.1. Valor presente de anualidades ordinarias con pagos fracccionados


1
Anualidades temporales n años. Consiste en una serie de n × m pagos, cada uno de $ m.
Respecto al momento en que se hace cada pago tenemos:

• Anualidad inmediata anticipada, el primer pago se realiza en la primera fracción del


primer periodo de pago y todos los pagos se hacen al principio de cada fracción de
(m)
periodo de pago. Su notación es: ä . El valor presente es:
n|

à !
(m) 1 1 2 1 1 mn−1
X t 1 1 − vn 1 − vn d
ä = (1+v m +v m +. . .+v n− m ) = vm = 1 = (m)
= (m) än|
n| m m t=0 m 1 − vm d d

• Anualidad inmediata vencida, el primer pago se realiza en la primera fracción del


primer periodo de pago y todos los pagos se hacen al final de cada fracción de periodo
(m)
de pago. Su notación es: a . El valor presente es:
n|

mn
(m) 1 1 2
n 1 X t 1 1 1 − vn 1 − vn i
a = (v + v + . . . + v ) =
m m vm = vm 1 = (m)
= (m) an|
n| m m t=1 m 1 − vm i i

• Anualidad diferida h años anticipada, el primer pago se realiza en la primera fracción


del periodo de pago h + 1 y todos los pagos se hacen al principio de cada fracción de
(m)
cada periodo de pago. Su notación es: h| ä . El valor presente es:
n|

m(h+n)−1
(m) 1 1 2 1 1 X 1 1 mn−1
X t (m)
h| än| = (v h +v h+ m +v h+ m +. . .+v h+n− m ) = vm = v h+ m = v h ä
m m t=mh
m t=0 n|
Matemáticas Financieras. Mercedes Gregorio 63

• Anualidad diferida h años vencida, el primer pago se realiza en la primera fracción


del periodo de pago h + 1 y todos los pagos se hacen al final de cada fracción de cada
(m)
periodo de pago. Su notación es: h| a . El valor presente es:
n|

mn m(h+n)
(m) 1 h+ 1 2 3 1 X t 1 X t (m)
h| a = (v m +v h+ m +v h+ m +. . .+v h+n ) = vm = v h+ m = v h a
n| m m t=mh+1 m t=1 n|

Perpetuidades

• Perpetuidad inmediata anticipada, el primer pago se realiza en la primera fracción del


primer periodo de pago y todos los pagos se hacen al principio de cada fracción de
(m)
cada periodo de pago. Su notación es: ä . El valor presente es:
∞|

(m) (m) 1 − vn 1
ä = lı́m ä = lı́m (m)
= (m)
∞| n→∞ n| n→∞ d d

• Perpetuidad inmediata vencida, el primer pago se realiza en la primera fracción del


primer periodo de pago y todos los pagos se hacen al final de cada fracción de cada
(m)
periodo de pago. Su notación es: a . El valor presente es:
∞|

(m) (m) 1 − vn 1
ä = lı́m ä = lı́m = (m)
∞| n→∞ n| n→∞ i(m) i

• Perpetuidad diferida h años anticipada, el primer pago se realiza en la primera fracción


del periodo de pago h + 1 y todos los pagos se hacen al principio de cada fracción de
(m)
cada periodo de pago. Su notación es: h| ä . El valor presente es:
∞|

µ ¶
(m) (m) (m) 1
h| ä∞| = lı́m ä = lı́m v h ä =v h
n→∞ h| n| n→∞ n| d(m)

• Perpetuidad diferida h años vencida, el primer pago se realiza en la primera fracción


del periodo de pago h + 1 y todos los pagos se hacen al final de cada fracción de cada
(m)
periodo de pago. Su notación es: h| a . El valor presente es:
∞|

µ ¶
(m) (m) (m) 1
h| a = lı́m h| a = lı́m v h a = vh
∞| n→∞ n| n→∞ n| i(m)

Estas anualidades también pueden manejarse con una tasa de interés efectiva por cada m-
ésimo y tratar los pagos como $1 por periodo.

Ejemplo 4.4 Un individuo pide prestados $5000.00. El reembolso se hace de la forma siguiente:
se realiza el primer pago al final del noveno mes y debe hacer 60 pagos mensuales iguales. Si la
tasa de interés es del 18 % anual convertible mensualmente.
64 Capítulo 4. Anualidades

1. Calcula la cantidad de cada pago

··· X ··· X X

8 9 67 68
0 ··· 12 12 ··· 12 12

5000

5000 = 12X 8 | a5| = 12X 9 | ä5|


12 12

5000 5000
⇒X= = 8 = 143.03
12 8 | a5| 12v 12
1−v 5
(12)
12 i

2. Calcula la cantidad de cada pago si se realizan los pagos de forma anticipada y sin diferi-
miento.

X X X X ··· X

1 2 3 59 60
0 12 12 12 ··· 12 12

5000

5000 = 12Xä5|

5000 5000 5000


⇒X= = 1−v5 = 5 = 125.09
12ä5| 12 d(12) 12 1−v
i(12)
(12)
1+ i 12
Matemáticas Financieras. Mercedes Gregorio 65

4.3.2. Monto de anualidades ordinarias con pagos fraccionados

No hay diferencias entre el monto de anualidades diferidas y no diferidas. Así como tampoco
existen montos de perpetuidades.

1
Anualidades temporales n años. Consiste en una serie de n × m pagos de $ m cada pago.
Respecto al momento en que se hace cada pago tenemos:

• Anualidad inmediata (o diferida) anticipada, el primer pago se realiza en la primera


fracción del primer periodo de pago y todos los pagos se hacen al principio de cada
(m)
fracción de cada periodo de pago. Su notación es: s̈ . El valor futuro es:
n|

nm
(m) 1 1 1 1 X t
s̈ = ((1 + i)n + (1 + i)n− m + . . . + (1 + i) m ) = (1 + i) m
n| m m t=1
n µ ¶
1 1 1 − (1 + i) (1 + i)n − 1 d
= (1 + i) m 1 = = s̈n|
m 1 − (1 + i) m d(m) d(m)

• Anualidad inmediata (o diferida) vencida, el primer pago se realiza en la primera


fracción del primer periodo de pago y todos los pagos se hacen al final de cada fracción
(m)
de cada periodo de pago. Su notación es: s . El valor presente es:
n|

(m) 1 1 2 1 nm−1
X t
s = ((1 + i)n− m + (1 + i)n− m + . . . + 1) = (1 + i) m
n| m m t=0
µ ¶
1 1 − (1 + i)n (1 + i)n − 1 i
= 1 = = sn|
m 1 − (1 + i) m i(m) i(m)

4.4. Anualidades continuas

Una anualidad continua ordinaria se define como una anualidad que paga de forma continua
a la tasa de $1 anual. Se puede ver como el límite de una anualidad fraccionada cuando el número
de pagos que se hacen en un año, m, tiende a infinito.
En este tipo de anualidades los valores presentes de anualidades anticipadas y vencidas coin-
ciden.
La notación que se utiliza para este tipo de anualidades es la siguiente. Para el valor presente
66 Capítulo 4. Anualidades

anualidad continua
?

t|ān|
diferida t unidades 6 6 duración de la anualidad
de tiempo (medida en unidades de tiempo)
(no se pone si es inmediata)

Para el monto o valor futuro:

anualidad continua
?

s̄n|
6 duración de la anualidad
(medida en unidades de tiempo)

4.4.1. Valor presente de anualidades ordinarias continuas


Anualidades temporales n años. En flujos de efectivo continuos a la tasa de $1 por año ($1
distribuido uniformemente de forma continua a lo largo de un año).

• Anualidad inmediata, se empieza a pagar en el primer instante del primer periodo de


pago. Su notación es: ān| . El valor presente es:

Z n " #n
vt 1 − vn
ān| = v t dt = =
0 ln v 0
δ
Z n " #n
e −δt 1 − e−δn
= e−δt dt = =
0 δ 0
δ

O bien, podemos calcular el límite de una anualidad ordinaria fraccionada cuando m


tiende a infinito:

(m) 1 − vn 1 − vn
ān| = lı́m ä = lı́m =
m→∞ n| m→∞ d(m) δ
Matemáticas Financieras. Mercedes Gregorio 67

(m) 1 − vn 1 − vn
ān| = lı́m a = lı́m =
m→∞ n| m→∞ i(m) δ

• Anualidad diferida h años, se empieza a pagar en el primer instante del periodo de


pago m + 1. Su notación es: m| ān| . El valor presente es:

Z h+n " #h+n


vt 1 − vn
h| ān| = v t dt = = vh
h ln v h
δ
Z h+n " #h+n
e −δt 1 − e−δn
= e−δt dt = = e−δh
h δ h
δ

O bien, podemos calcular el límite de una anualidad ordinaria fraccionada cuando m


tiende a infinito:

(m) 1 − vn h1 − v
n
h| ān| = lı́m h| ä = lı́m v h = v
m→∞ n| m→∞ d(m) δ
1 − v n 1 − vn
(m)
h| ān| = lı́m a = lı́m v h (m) = v h
m→∞ h| n| m→∞ i δ

Perpetuidades

• Perpetuidad inmediata, se empieza a pagar en el primer instante del primer periodo


de pago. Su notación es: ā∞| . El valor presente es:

1 − vn 1
ā∞| = lı́m ān| = lı́m =
n→∞ n→∞ δ δ
• Perpetuidad diferida h años, se empieza a pagar en el primer instante del periodo de
pago m + 1. Su notación es: m| ā∞| . El valor presente es:
µ ¶
h h 1
h| ā∞| = lı́m
n→∞ h| n|
ā = lı́m v ān| = v
n→∞ δ

4.4.2. Monto de anualidades ordinarias continua


No hay diferencias entre el monto de anualidades diferidas y no diferidas. Así como tampoco
existen montos de perpetuidades.
Anualidades temporales n años.
• Anualidad inmediata (o diferida). Su notación es: s̄n| . El valor futuro es:

Z n " #n
t (i + i)t (1 + i)n − 1
s̄n| = (1 + i) dt = =
0 ln(1 + i) 0
δ
Z n " #n
eδt eδn − 1
= eδt dt = =
0 δ 0
δ
68 Capítulo 4. Anualidades

O bien, podemos calcular el límite del monto de una anualidad ordinaria fraccionada
cuando m tiende a infinito:

(m) (1 + i)n − 1 (1 + i)n − 1


s̄n| = lı́m s̈ = lı́m =
m→∞ n| m→∞ d(m) δ
(m) (1 + i)n − 1 (1 + i)n − 1
s̄n| = lı́m s = lı́m =
m→∞ n| m→∞ i(m) δ

4.4.3. Fuerza de interés variable


Si la fuerza de interés es variable entonces:

Z n Rt
ān|δ(t) = e− 0
δ(s)ds
dt
0
Z n Rn
δ(s)ds
s̄n|δ(t) = e t dt
0
n−1
X Rt
− δ(s)ds
än|δ(t) = e 0

t=0
n−1
X Rn
δ(s)ds
s̈n|δ(t) = e t

t=0
n
X Rt
− δ(s)ds
an|δ(t) = 0e
t=1
Xn Rn
δ(s)ds
sn|δ(t) = e t

t=1

4.5. Anualidades variables


En el caso general, donde no todos los pagos son iguales, se paga la cantidad Cs al tiempo
ts , s = 1, . . . , n.
El valor presente de esta anualidad está dada por
n
X
VP = C s v ts
s=1

y el valor futuro o monto al tiempo del último pago es


n
X
VF = Cs (1 + i)tn −ts .
s=1

Se pueden estudiar anualidades donde la variación de los pagos es común. Los más comunes
tipos de variación son: variación mediante progresión aritmética o mediante progresión aritmética.
Matemáticas Financieras. Mercedes Gregorio 69

4.5.1. Anualidades con variación anual y pagos anuales


4.5.1.1. Variación mediante progresión aritmética
Una progresión aritmética es una sucesión de términos que tienen una diferencia constante.
Si se designa el primer término por a (valor inicial) y la diferencia por d (tasa de crecimiento) la
progresión será: a, a + d, a + 2d, a + 3d, . . . .
Si se considera una anualidad temporal por n años con pagos anuales, inmediata, vencida,
donde el primer pago es C y crecen en una progresión aritmética de razón R por año

C + (n − 1)R
C C +R C + 2R C + (n − 2)R

0 1 2 3 ··· n−1 n

su valor presente está dado por:

V P = Cv + (C + R)v 2 + (C + 2R)v 3 + . . . + (C + (n − 2)R)v n−1 + (C + (n − 1)R)v n ,

si multiplicamos ambos lados de la ecuación por (1 + i), entonces:

(1 + i)V P = C + (C + R)v + (C + 2R)v 2 + . . . + (C + (n − 2)R)v n−2 + (C + (n − 1)R)v n−1 .

Si restamos estas dos ecuaciones, tenemos:

(1 + i)V P − V P = iV P = C + Rv + Rv 2 + . . . + Rv n−2 + Rv n−1 − (C + (n − 1)R)v n

sumamos y restamos el término Rv n

(1 + i)V P − V P = iV P = C + Rv + Rv 2 + . . . + Rv n−2 + Rv n−1 + Rv n − (C + (n − 1)R)v n − Rv n

= C + Ran| − Cv n − nRv n = C(1 − v n ) + R(an| − nv n )


despejamos el valor de V P
µ ¶ Ã ! Ã !
1 − vn an| − nv n an| − nv n
VP =C +R = Can| + R .
i i i

Procediendo de forma similar, el valor futuro o monto al final del periodo donde se realiza el
último pago es
à !
sn| − n
V F = Csn| + R .
i
70 Capítulo 4. Anualidades

4.5.1.1.1. Anualidades crecientes ordinarias (progresión aritmética) Si se consideran


pagos anuales en una progresión aritmética con C = R = 1 obtenemos una anualidad creciente
ordinaria.
La notación que se utiliza para considerar pagos anuales crecientes de forma anual se incluye
una I al lado izquierdo de los símbolos de valores presentes y montos.
Valor presente de anualidades temporales n años, pagos anuales
Anualidad creciente ordinaria inmediata vencida
à !
an| − nv n ian| + an| − nv n än| − nv n
(Ia)n| = Can| + R = =
i i i

Anualidad creciente ordinaria inmediata anticipada


än| − nv n
(Iä)n| = (1 + i)(Ia)n| = = än| + (Ia)n−1|
d
Anualidad creciente ordinaria diferida h años vencida

h| (Ia)n| = v h (Ia)n|

Anualidad creciente ordinaria diferida h años anticipada

h| (Iä)n| = v h (Iä)n|

El valor presente de las perpetuidades crecientes


Perpetuidad creciente ordinaria inmediata vencida
än| − nv n 1
1+i
(Ia)∞| = lı́m (Ia)n| = lı́m = d = 2
n→∞ n→∞ i i i
Perpetuidad creciente ordinaria inmediata anticipada
än| − nv n 1
1
(Iä)∞| = lı́m (Iä)n| = lı́m = d = 2
n→∞ n→∞ d d d
Perpetuidad creciente ordinaria diferida h años vencida

h| (Ia)∞| = v h (Ia)∞|

Perpetuidad creciente ordinaria diferida h años anticipada

h| (Iä)∞| = v h (Iä)∞|

Y para los valores futuros o montos


Anualidad creciente ordinaria vencida
ä − nv n s̈n| − n
n n n|
(Is)n| = (1 + i) (Ia)n| = (1 + i) =
i i
Anualidad creciente ordinaria vencida
än| − nv n s̈n| − n
(I s̈)n| = (1 + i)n (Iä)n| = (1 + i)n =
d d
Matemáticas Financieras. Mercedes Gregorio 71

4.5.1.1.2. Anualidades decrecientes ordinarias (progresión aritmética) Considere-


mos ahora anualidades que decrecen en progresión aritmética desde n hasta 1; C = n, R = −1.
La notación que se utiliza para considerar pagos anuales decrecientes de forma anual se incluye
una D al lado izquierdo de los símbolos de valores presentes y montos.
Valor presente de anualidades temporales n años, pagos anuales

Anualidad decreciente ordinaria inmediata vencida


à !
an| − nv n inan| − an| + nv n n − an|
(Da)n| = Can| + R = =
i i i

Anualidad decreciente ordinaria inmediata anticipada

n − an|
Dn| = (1 + i)(Da)n| =
d

Anualidad decreciente ordinaria diferida h años vencida

h| (Da)n| = v h (Da)n|

Anualidad decreciente ordinaria diferida h años anticipada

h| (Dä)n| = v h (Dä)n|

No existen perpetuidades decrecientes.


Y para los valores futuros o montos

Anualidad decreciente ordinaria vencida

n − an| n(1 + i)n − sn|


n n
(Ds)n| = (1 + i) (Da)n| = (1 + i) =
i i

Anualidad decreciente ordinaria vencida

n − an| n(1 + i)n − sn|


n n
(Ds̈)n| = (1 + i) (Dä)n| = (1 + i) =
d d

4.5.1.2. Variación mediante progresión geométrica

Una progresión geométrica es una sucesión de términos en donde cada término puede obte-
nerse del anterior multiplicándolo por una cantidad constante llamada razón. Si se designa el
primer término por a (valor inicial) y la razón por r, la progresión será: a, ar, ar2 , ar3 , . . ..
Si se considera una anualidad temporal por n años pagos anuales, inmediata, vencida, donde
el primer pago es a y crecen en una progresión aritmética de razón r por año
72 Capítulo 4. Anualidades

a ar ar2 arn−2 arn−1

0 1 2 3 ··· n−1 n

su valor presente está dado por:


Si r 6= 1 + i

VP = av ³+ arv 2 + ar2 v 3 + . . . + arn−2 v n−1 + arn−1´v n


= av 1 + rv + (rv)2 + . . . + (rv)n−2 + (rv)n−1
µ ¶
1 − (rv)n
= av
1 − rv
µ ¶
1 − (rv)n
= a
1+i−r
si r = 1 + i

V P = nav

4.5.1.2.1. Anualidades ordinarias (progresión geométrica) Consideremos a = 1, si


0 < r < 1 tenemos una anualidad decreciente, mientras que si 1 < r tenemos una anualidad
creciente.
La notación para este tipo de anualidades es anteponer una V al símbolo de valor presente o
valor futuro de la anualidad.
Valor presente de anualidades temporales n años, pagos anuales

Anualidad ordinaria inmediata vencida


(
1−(rv)n
(1+i)−r r=6 1+i
(V a)n| =
nv r =1+i

Anualidad ordinaria inmediata anticipada


(
1−(rv)n
r=6 1+i
(V ä)n| = (1 + i)(V a)n| = 1−rv
n r =1+i

Anualidad ordinaria diferida h años vencida

h| (V a)n| = v h (V a)n|
Matemáticas Financieras. Mercedes Gregorio 73

Anualidad ordinaria diferida h años anticipada

h| (V ä)n| = v h (V ä)n|

El valor presente de las perpetuidades crecientes. Sólo se puede calcular si rv < 1

Perpetuidad ordinaria inmediata vencida

1 − (rv)n 1
(V a)∞| = lı́m (V a)n| = lı́m =
n→∞ n→∞ (1 + i) − r (1 + i) − r

Perpetuidad creciente ordinaria inmediata anticipada

1 − (rv)n 1
(V ä)∞| = lı́m (V ä)n| = lı́m =
n→∞ n→∞ 1 − rv 1 − rv

Perpetuidad ordinaria diferida h años vencida

h| (V a)∞| = v h (V a)∞|

Perpetuidad ordinaria diferida h años anticipada

h| (V ä)∞| = v h (V ä)∞|

Y para los valores futuros o montos

Anualidad ordinaria vencida


( (
1−(rv)n rn −(1+i)n
n n (1+i)−r r=6 1+i r−(1+i) r=6 1+i
(V s)n| = (1 + i) (V a)n| = (1 + i) =
nv r =1+i n(1 + i)n−1 r =1+i

Anualidad ordinaria vencida


( (
1−(rv)n rn −(1+i)n
n n r=6 1+i rv−1) r=6 1+i
(V s̈)n| = (1 + i) (V ä)n| = (1 + i) 1−rv =
n r =1+i n(1 + i)n r =1+i

4.5.1.3. Ejemplos

Ejemplo 4.5 Un padre invierte en el 1er. cumpleaños de su hijo $1000.00, $2000.00 en el se-
gundo cumpleaños, $3000.00 en el tercero, etc. hasta el cumpleaños 21.
Al cumplir 21 años, ¿cuánto recibirá el hijo si la tasa de interés a la que fueron invertidos
los depósitos es de 7 % efectivo anual?
74 Capítulo 4. Anualidades

=1000[ 1 2 3 20 21]

1000 2000 3000 20000 21000

0 1 2 3 ··· 20 21

   
à ! (1.07)2 1−1
s̈21| − 21 (1+i)2 1−1
− 21 0.07 − 21
1000(Is)21| = 1000 = 1000  d  = 1000 
 1.07 
 = 385, 796.27
i i 0.07

Ejemplo 4.6 Se adquiere una deuda por $100,000.00 que deberán pagarse mediante pagos anua-
les al final de cada año durante 10 años. Los pagos se incrementarán a una tasa del 4 % cada
año, el primer pago es de X. Si la tasa de interés efectiva anual es del 7 %. ¿Cuál es el pago
inicial X?

= X[ 1 1.041 1.042 1.048 1.049 ]

X X(1.04) X(1.04)2 X(1.04)8 X(1.04)9

0 1 2 3 ··· 9 10

r = 1.04, 1 + i = 1.07
 ³ ´10 
1
 1 − 1.04 1.07 
100, 000 = X(V a)10| = X   = X(8.2506)
1.07 − 1.04

100, 000
⇒X= = 12, 120.29
8.2506
Ejemplo 4.7 Se van a hacer 5 pagos, el primero dentro de 5 años de $10,000, el siguiente un
año después de $9,000, un año después de $8,000, así sucesivamente hasta el último pago que es
de $6,000. ¿Cuál es el valor presente de esos pagos, a una tasa efectiva anual del 10 %?
Matemáticas Financieras. Mercedes Gregorio 75

5 5 5 5 5
=1,000[ + + + + +]
5 4 3 2 1
10,0009,000 8,000 7,000 6,000

0 1 2 3 4 5 6 7 8 9 10

10005| (Dä)5| + 50005| ä5| = 10004| (Da)5| + 50004| a5|


à à !! à !
5 − a5| 1 − v5 1
1 − 1.11 5 1
5 1.15
= 1000v +5 = 1000 0.1 5− +5−5 5 = 21, 204.88
d d 1.1
0.1 1.1
También podría resolverse el problema completando la serie y restando los elementos que se
utilizaron para completar la serie

-5,000-4,000-3,000-2,000-1,000

10,0009,000 8,000 7,000 6,000 5,000 4,000 3,000 2,000 1,000

0 1 2 3 4 5 6 7 8 9 10 11 12 13 14 15

10005| (Dä)10| − 100010| ä5| = 21, 204.88


No perdiendo de vista que puede también calcularse como

10, 000v 5 + 9, 000v 6 + 8, 000v 7 + 7, 000v 8 + 6, 000v 9 = 21, 204.88

4.5.2. Anualidades con variación anual y pagos fraccionados


Vamos a considerar en esta sección anualidades que hacen m pagos al año y la cuantía de
estos pagos se mantiene constante durante el año y al final del año se varía la cuantía. El valor
m puede ser tanto menor como mayor a uno; m > 1 implica pagos con mayor frecuencia que la
anual, mientras m < 1 implica pagos con menor frecuencia que la anual. En el desarrollo que se
presenta vamos a considerar que m > 1 pero se puede verificar que el resultado es el mismo para
m < 1.
76 Capítulo 4. Anualidades

4.5.2.1. Variación mediante progresión aritmética


Consideremos una anualidad que realiza pagos m veces al año de forma inmediata y pagos
C
vencidos donde el primer pago es m y los pagos varían una vez al año de acuerdo a una progresión
R
aritmética con razón m .

C C C C C+R C+R C+R C+2R C+(n−1)R C+(n−1)R


m m ··· m m m ··· m m m ··· m m

1 2 m−1 m+1 2m−1 2m+1 nm−1


0 m m ··· m 1 m ··· m 2 m ··· m n

su valor presente está dado por:

C 1 C 2 C C + R 1+ 1 C + R 1+ 2 C +R 2
VP = vm + vm + ... + v + v m+ v m + ... + v + ...
m m m m m m
C + (n − 1)R n−1+ 1 C + (n − 1)R n
+ v m + ... + v
m m
1 C C 1 C 1 C + R + C + R 1+ 1 C + R 2− 1
(1 + i) m V P = + v m + . . . + v 1− m + v v m + ... + v m + ...
m m m m m m
C + (n − 1)R n−1 C + (n − 1)R n− 1
+ v + ... + v m
m m

Restamos estas dos ecuaciones, tenemos:

1 C R R R R C + (n − 1)R n
(1 + i) m V P − V P + v + v 2 + . . . + v n−2 + v n−1 −
= v
m m m m m m
1 1 h i
((1 + i) m − 1)V P = C(1 − v n ) + R(an| − nv n )
m
despejamos el valor de V P
h i
C(1 − v n ) + R(an| − nv n ) C(1 − v n ) + R(an| − nv n )
VP = 1 =
m((1 + i) m − 1) i(m)

4.5.2.1.1. Anualidad creciente ordinaria En este caso C = R = 1


Valor presente de la anualidad temporal n años (nm pagos)
Anualidad creciente ordinaria inmediata vencida
(m)
1 − v n + an| − nv n än| − nv n
(Ia) = =
n| i(m) i(m)
Matemáticas Financieras. Mercedes Gregorio 77

Anualidad creciente ordinaria inmediata anticipada

(m) 1 (m)
än| − nv n än| − nv n än| − nv n
(Iä) = (1 + i) m (Ia) = 1 =³ ´ µ³ ´−1 ¶ =
n| n|
v m i(m) d(m) d(m) d(m)
1− m m 1− m −1

Anualidad creciente ordinaria diferida vencida


(m) (m)
k| (Ia)n| = v k (Ia)
n|

Anualidad creciente ordinaria diferida anticipada


(m) (m)
k| (Iä)n| = v k (Iä)
n|

Para el valor presente de las perpetuidades

Perpetuidad creciente ordinaria inmediata vencida

(m) (m)
än| − nv n 1 1
(Ia) = lı́m (Ia) = lı́m =
∞| n→∞ n| n→∞ i(m) i(m) d

Perpetuidad creciente ordinaria inmediata anticipada

(m) (m)
än| − nv n 1 1
(Iä) = lı́m (Iä) = lı́m =
∞| n→∞ n| n→∞ d(m) d(m) d

Para el monto de las anualidades

Anualidad creciente ordinaria inmediata vencida

(m) (m)
än| − nv n s̈n| − n
(Is) = (1 + i)n (Ia) = (1 + i)n =
n| n| i(m) i(m)

Anualidad creciente ordinaria inmediata anticipada


ä − nv n s̈n| − n
(m) n (m) n n|
(I s̈) = (1 + i) (Iä) = (1 + i) =
n| n| d(m) d(m)

4.5.2.1.2. Anualidad decreciente ordinaria En este caso C = n, R = −1


Valor presente de la anualidad temporal n años (nm pagos)
Anualidad decreciente ordinaria inmediata vencida

(m)
n(1 − v n ) − an| + nv n n − an|
(Da) = =
n| i(m) i(m)

Anualidad decreciente ordinaria inmediata anticipada

(m) 1 (m)
n − an| n − an|
(Dä) = (1 + i) m (Da) = 1 =
n| n|
v i(m)
m d(m)
78 Capítulo 4. Anualidades

Anualidad decreciente ordinaria diferida vencida


(m) (m)
k| (Da)n| = v k (Da)
n|

Anualidad decreciente ordinaria diferida anticipada


(m) (m)
k| (Dä)n| = v k (Dä)
n|

No existe el valor presente de las perpetuidades.


Para el monto de las anualidades

Anualidad decreciente ordinaria inmediata vencida

(m) (m)
n − an| n(1 + i)n − sn|
n n
(Ds) = (1 + i) (Da) = (1 + i) =
n| n| i(m) i(m)

Anualidad decreciente ordinaria inmediata anticipada

(m) (m)
n − an| n(1 + i)n − sn|
(Ds̈) = (1 + i)n (Dä) = (1 + i)n =
n| n| d(m) d(m)

4.5.2.2. Variación mediante progresión geométrica

a a a a ar ar ar ar 2 ar n−1 ar n−1
m m ··· m m m ··· m m m ··· m m

1 2 m−1 m+1 2m−1 2m+1 nm−1


0 m m ··· m 1 m ··· m 2 m ··· m n

Su valor presente es

a 1 a 2 a ar 1 ar 2 ar
VP = v m + v m + . . . + v + v 1+ m + v 1+ m + . . . + v 2 + . . .
m m m m m m
arn−1 n−1+ 1 arn−1 n−1+ 2 arn−1 n
+ v m + v m + ... + v
m m m
a 1 2 ar 1 2 arn−1 n−1 1 2
= (v m + v m + . . . + v) + v(v m + v m + . . . + v) + . . . + v (v m + v m + . . . + v)
m m m
1 1 2
2 n−1
= a (v m + v m + . . . + v)(1 + (rv) + (rv) + . . . + (rv) )
m
Si 1 + i 6= r
Matemáticas Financieras. Mercedes Gregorio 79

µ ¶ µ ¶µ ¶
(m) 1 − (rv)n d 1 − (rv)n
VP = aa =a
1| 1 − rv i(m) 1 − rv
Si 1 + i = r
µ ¶
(m) d
VP = aa n = na
1| i(m)

4.5.2.2.1. Anualidades ordinarias variables (progresión geométrica) Consideremos


a = 1, si 0 < r < 1 tenemos una anualidad decreciente, mientras que si 1 < r tenemos una
anualidad creciente.
La notación para este tipo de anualidades es anteponer una V al símbolo de valor presente o
valor futuro de la anualidad.
Valor presente de anualidades temporales n años, pagos m veces al año, constantes durante
el año

Anualidad ordinaria variables inmediata vencida


 ³ ´
 d 1−(rv)n
(m) i(m) 1−rv r=6 1+i
(V a) = d
n|  n r =1+i
i( m)

Anualidad ordinaria variables inmediata anticipada


 ³ ´
 d 1−(rv)n
(m) 1 (m) d(m) 1−rv r=6 1+i
(V ä) = (1 + i) (V a)m = d
n| n|  n r =1+i
d( m)

Anualidad ordinaria variables diferida h años vencida

(m) (m)
h| (V a) = v h (V a)
n| n|

Anualidad ordinaria variables diferida h años anticipada

(m) (m)
h| (V ä) = v h (V ä)
n| n|

El valor presente de las perpetuidades variables. Sólo se puede calcular si rv < 1

Perpetuidad ordinaria variables inmediata vencida

(m) (m) d 1 − (rv)n d 1


(V a) = lı́m (V a) = lı́m = (m)
∞| n→∞ n| n→∞ i(m) 1 − rv i 1 − rv

Perpetuidad ordinaria variables inmediata anticipada

(m) (m) d 1 − (rv)n d 1


(V ä) = lı́m (V ä) = lı́m = (m)
∞| n→∞ n| n→∞ d(m) 1 − rv d 1 − rv
80 Capítulo 4. Anualidades

Perpetuidad ordinaria variables diferida h años vencida

(m) (m)
h| (V a) = v h (V a)
∞| ∞|

Perpetuidad ordinaria variables diferida h años anticipada

(m) (m)
h| (V ä) = v h (V ä)
∞| ∞|

Y para los valores futuros o montos


Anualidad ordinaria variables vencida
( (
d 1−(rv)n d (1+i)n −rn
(m) n (m) n i(m) 1−rv r=6 1+i i(m) 1−rv r 6= 1 + i
(V s) = (1+i) (V a) = (1+i) d = d
n| n|
i(m)
n r =1+i i (m) n(1 + i)n r =1+i

Anualidad ordinaria variables anticipada


( (
d 1−(rv)n d (1+i)n −rn
(m) n (m) n d(m) 1−rv r=6 1+i d(m) 1−rv r=6 1+i
(V s̈) = (1+i) (V ä) = (1+i) d = d
n| n|
d(m)
n r =1+i d(m) n(1 + i)n r =1+i

4.5.3. Anualidades con variación fraccionaria y pagos fraccionados


Se van a considerar anualidades donde se realizan pagos con frecuencia distinta a la anual y la
variación en los pagos también se realiza con una frecuencia distinta a la anual. No necesariamente
la frecuencia de pagos será igual a la frecuencia de variación en ellos. Sea m el número de pagos
que se realizan en un año y sea p el número de veces que los pagos varían a lo largo del año.
Al igual que en los casos anteriores la variación en los pagos puede darse mediante progresión
aritmética o mediante progresión geométrica. Las variables m y p deben satisfacer que una sea
múltiplo de la otra porque en caso contrario no tendríamos un patrón en los pagos y el cálculo
de valores presentes y montos debería hacerse de forma manual. En los gráficos que vamos a
desarrollar supondremos que m y p son mayores a 1 y m es múltiplo de p; pero el desarrollo es
válido para valores menores a uno.

4.5.3.1. Variación mediante progresión aritmética


Consideremos una anualidad que realiza pagos m veces al año de forma inmediata y pagos
C
vencidos donde el primer pago es mp y los pagos varían p veces al año de acuerdo a una progresión
R
aritmética con razón mp .

C C+R C+(p−1)R C+pR C+(2p−1)R C+(np−2)R C+(np−1)R


mp mp mp mp mp mp mp
··· ··· ···
C C+R C+(p−1)R C+pR C+(2p−2)R C+(2p−1)R C+(np−1)R
mp mp mp mp mp mp mp

1 2 3 4 m−1 m+1 m+2 2m−2 2m−1 nm−2nm−1


0 m m m m ··· m 1 m m ··· m m 2 ··· m m n
1 2 p+1 2p−1 np−1
0 p p 1 p ··· p 2 ··· p n
Matemáticas Financieras. Mercedes Gregorio 81

su valor presente está dado por:

C 1 C 2 C +R 3 C +R 4 C + (p − 1)R 1− 1
VP = vm + vm + vm + vm + ... + v m
mp mp mp mp mp
C + (p − 1)R C + pR 1+ 1 C + pR 1+ 2 C + (2p − 2)R 2− 2
+ v+ v m+ v m + ... + v m
mp mp mp mp
C + (2p − 1)R 2− 1 C + (2p − 1)R 2
+ v m+ v + ...
mp mp
C + (np − 2)R n− 2 C + (np − 1)R n− 1 C + (np − 1)R n
+ v m+ v m+ v
mp mp mp
1 C C 1 C +R 2 C +R 3 C + (p − 1)R 1− 2
(1 + i) m V P = + vm + vm + vm + ... + v m
mp mp mp mp mp
C + (p − 1)R 1− 1 C + pR C + pR 1+ 1 C + (2p − 2)R 2− 3
+ v m+ v+ v m + ... + v m
mp mp mp mp
C + (2p − 1)R 2− 2 C + (2p − 1)R 2− 1
+ v m+ v m + ...
mp mp
C + (np − 2)R n− 3 C + (np − 1)R n− 2 C + (np − 1)R n− 1
+ v m+ v m+ v m
mp mp mp

Restamos estas dos ecuaciones, tenemos:

1 C R 2 R 4 R n− 4 R n− 2 C + (np − 1)R n
(1 + i) m V P − V P = + vm + vm + ... + v m+ v m− v
mp mp mp mp mp mp
1 C R p1 R p2 R n− p2 R n− p1 C + (np − 1)R n
(1 + i) m V P − V P = + v + v + ... + v + v − v
mp mp mp mp mp mp
· ¸
1 1 C (p)
((1 + i) m − 1)V P = (1 − v n ) + R(a − nv n )
m p n|

despejamos el valor de V P
· ¸
C (p) (p)
p (1 − vn) + R(a − nv n ) C
p (1 − v n ) + R(a − nv n )
n| n|
VP = 1 =
m((1 + i) m − 1) i(m)

4.5.3.1.1. Anualidad creciente ordinaria En este caso C = R = 1


Valor presente de la anualidad temporal n años (nm pagos)
Anualidad creciente ordinaria inmediata vencida
1 (p) (p)
p − p1 v n + a − nv n ä − nv n
(p) (m) n| n|
(I a) = =
n| i(m) i(m)
Anualidad creciente ordinaria inmediata anticipada
(p)
ä − nv n
(p) (m) 1
(p) (m) n|
(I ä) = (1 + i) (I m a) =
n| n| d(m)
82 Capítulo 4. Anualidades

Anualidad creciente ordinaria diferida vencida


(p) (m) (m)
k| (I a) = v k (I (p) a)
n| n|

Anualidad creciente ordinaria diferida anticipada


(p) (m) (m)
k| (I ä) = v k (I (p) ä)
n| n|

Para el valor presente de las perpetuidades

Perpetuidad creciente ordinaria inmediata vencida


(p)
ä − nv n 1 1
(m) (m) n|
(I (p) a) = lı́m (I (p) a) = lı́m =
∞| n→∞ n| n→∞ i(m) i(m) d(p)

Perpetuidad creciente ordinaria inmediata anticipada


(p)
ä − nv n 1 1
(p) (m) (p) (m) n|
(I ä) = lı́m (I ä) = lı́m =
∞| n→∞ n| n→∞ d(m) d(m) d(p)

Para el monto de las anualidades

Anualidad creciente ordinaria inmediata vencida


(p) (p)
ä − nv n s̈ − n
(p) (m) n (p) (m) n n| n|
(I s) = (1 + i) (I a) = (1 + i) =
n| n| i(m) i(m)

Anualidad creciente ordinaria inmediata anticipada


(p) (p)
ä − nv n s̈ −n
(m) (m) n n| n|
(I (p) s̈) = (1 + i)n (I (p) ä) = (1 + i) =
n| n| d(m) d(m)

4.5.3.1.2. Anualidad decreciente ordinaria En este caso C = np, R = −1


Valor presente de la anualidad temporal n años (nm pagos)
Anualidad decreciente ordinaria inmediata vencida
(p) (p)
n(1 − v n ) − a + nv n n−a
(p) (m) n| n|
(D a) = =
n| i(m) i(m)

Anualidad decreciente ordinaria inmediata anticipada


(p)
n−a
(m) 1 (m) n|
(D(p) ä) = (1 + i) m (D(p) a) =
n| n| d(m)

Anualidad decreciente ordinaria diferida vencida


(p) (m) (m)
k| (D a) = v k (D(p) a)
n| n|
Matemáticas Financieras. Mercedes Gregorio 83

Anualidad decreciente ordinaria diferida anticipada


(p) (m) (m)
k| (D ä) = v k (D(p) ä)
n| n|

No existe el valor presente de las perpetuidades.


Para el monto de las anualidades
Anualidad decreciente ordinaria inmediata vencida
(p) (p)
n−a n(1 + i)n − s
(p) (m) n (p) (m) n n| n|
(D s) = (1 + i) (D a) = (1 + i) =
n| n| i(m) i(m)
Anualidad decreciente ordinaria inmediata anticipada
(p) (p)
n−a n(1 + i)n − s
(p) (m) n (p) (m) n n| n|
(D s̈) = (1 + i) (D ä) = (1 + i) =
n| n| d(m) d(m)
Puede observarse que los casos de anualidades cuyos pagos varían de acuerdo a una progresión
aritmética vistos anteriormente son casos particulares de las anualidades presentadas en esta
sección estableciendo p = 1, m = 1 o ambos según sea el caso.

4.5.3.2. Variación mediante progresión geométrica


En el caso de que los pagos varían de acuerdo a una progresión geométrica con variación
diferente a la anual el encontrar una forma cerrada para los valores presentes y futuros se vuelve
demasiado complicado y es mejor el cómputo directo.

4.5.4. Anualidades variables continuas


El caso de las anualidades continuas, como ya se ha mencionado, puede verse como el límite
de las anualidades discretas cuando las variables correspondientes se hacen tender a infinito según
corresponda. Para el caso de anualidades continuas podemos considerar varios casos:
1. Variación de los pagos de forma fraccionada y pago de forma continua. Se considera el límite
cuando m tiende a infinito. Por ejemplo, consideremos el caso de una anualidad ordinaria
inmediata vencida temporal a n años cuyos pagos varían de acuerdo a una progresión
aritmética de forma fraccionada y los pagos se realizan de forma continua, entonces el
valor presente de esta anualidad será:
(p) (p)
ä − nv n ä − nv n
(p) (p) (m) n| n|
(I ā)n| = lı́m (I a) = lı́m = .
m→∞ n| m→∞ i(m) δ
2. Variación de los pagos de forma continua y pago de forma continua. Se considera el límite
cuando tanto p como m se hacen tender a infinito. Por ejemplo, consideremos el caso de una
anualidad ordinaria inmediata vencida temporal a n años cuyos pagos varían de acuerdo
a una progresión aritmética de forma continua y los pagos se realizan de forma continua,
entonces el valor presente de esta anualidad será:
(p)
ä − nv n ān| − nv n
¯ (p) (m) n|
(Iā) n|
= lı́m (I a) = lı́m = .
m→∞p→∞ n| m→∞p→∞ i(m) δ
84 Capítulo 4. Anualidades

4.6. Aplicaciones
Los problemas que pueden resolverse mediante anualidades son los que responden a las pre-
guntas de la figura 1.2 cuando alguna o todas las cantidades de interés se presentan en forma de
parcialidades y no en un único pago. La solución de este tipo de problemas como ya vimos pasa
por el planteamiento de la ecuación de valor y encontrar el (los) valor(es) faltante(s).

Ejemplo 4.8 Se va a comprar una casa que tiene un valor de $2,000,000. Se va a dar un
enganche del 20 % del costo y para pagar el resto se va a tomar un crédito a 20 años a una tasa
de interés efectivo anual fijo de r %. Se desea que el pago mensual sea de $15,000 o menos, ¿cuál
es la máxima tasa de interés que se puede aceptar?
(12)
12Xa = 2000000 − 2000000(0.20)
20|

2000000(0.80) 1600000
⇒X= = µ ¶ ≤ 15000.
(12) 1−(1+r)−20
12a 12
20| 1
12[(1+r) 12 −1]

r X
0.0900 14036.21
0.1000 14986.23
0.1001 14995.83
0.1002 15005.42
0.1005 15034.22
0.1010 15082.26
0.1025 15226.63
0.1050 15468.12
La máxima tasa de interés que se puede aceptar es el 10.01 % efectiva anual.

Ejemplo 4.9 Para pagar un préstamo de $10,000 se harán pagos anuales vencidos de 2000, 3000,
4000, 5000 durante los próximos 4 años. ¿Qué tasa de interés efectiva anual se está considerando?
Resuelve el problema utilizando el método de Newton-Raphson.

1000[a4| + (Ia)4| ] = 10000

1 − (1 + i)−4 (1 + i) + (1 + i)−3 4(1 + i)−4


f (i) = a4| + (Ia)4| − 10 = + − − 10
i i2 i

20(1 + i)−5 8(1 + i)−4 2[(1 + i) − (1 + i)−3 ]


f 0 (i) = + −
i i2 i3
Sea i0 = 0.10, entonces

f (i0 )
i1 = i0 − = 0.12680998
f 0 (i0 )
Matemáticas Financieras. Mercedes Gregorio 85

f (i1 )
i2 = i1 − = 0.12825342
f 0 (i1 )

f (i2 )
i3 = i2 − = 0.12825727
f 0 (i2 )

f (i3 )
i4 = i3 − = 0.12825727
f 0 (i3 )

La tasa que se está considerando es el 12.83 % efectiva anual.


86 Capítulo 4. Anualidades
Parte III

Amortizaciones

87
Capítulo 5

Amortización

5.1. Concepto de amortización


Amortización es un método para liquidar una deuda mediante pagos periódicos general-
mente iguales en los que se incluyen tanto intereses como capital.
Estos pagos forman una anualidad por lo tanto los problemas que involucran la amortización
de un adeudo son análogos a los problemas con anualidades.
En ocasiones, sin embargo, es necesario contar con un registro que indique periodo por periodo
la parte del pago que se aplica al pago de intereses y la parte que se destina para abonar parte del
capital, de esta manera se podrá decir de inmediato con qué suma de contado se podrá liquidar
el adeudo; además, se puede ver qué intereses han sido pagados en un periodo dado para poder
calcular las deducciones o impuestos que se deriven. Por ejemplo, para el pago de impuestos
los intereses reales pagados por créditos hipotecarios son deducibles de impuestos. Este registro
recibe el nombre de tabla de amortización.
Una tabla de amortización está constituida por columnas que muestran para cada periodo
de pago: el capital insoluto al principio del periodo (i.e. la deuda que se tiene en ese momento),
el pago a realizarse durante el periodo, los intereses contenidos en el pago, el capital contenido
en el pago.
La parte de la deuda no cubierta en una fecha dada se conoce como saldo insoluto o capital
insoluto. El capital insoluto al inicio del plazo es la deuda original y al final del plazo es cero (en
la práctica puede variar de cero por algunos centavos debido al redondeo utilizado en la tabla).
El pago a realizarse en un periodo dado está determinado por la forma en que se va a pagar
la deuda. Generalmente se construyen las tablas con pagos iguales y vencidos.
Los intereses contenidos en el pago, son los intereses que deben pagarse durante el periodo
por tener una deuda igual al capital insoluto al principio del periodo.

intereses contenidos en el pago

= (capital insoluto al principio del periodo) ∗ (tasa de interés efectiva por periodo)
El capital contenido en el pago es la diferencia del pago que se realiza y los intereses
que deben pagarse. Este capital se utiliza para reducir la deuda; se le resta al capital insoluto al
principio del periodo y se obtiene el capital insoluto para el inicio del siguiente periodo.

capital contenido en el pago = (pago) − (intereses contenidos en el pago )

89
90 Capítulo 5. Amortización

Tiempo Periodo Capital insoluto al Pago Interés contenido Capital contenido


t [t, t + 1) principio del periodo en el pago en el pago
0 1 C = Xan| X Xan| i X − X(1 − v n )
=X(1 − v n ) =Xv n
1 2 Xan| − Xv n X Xan−1| i X − X(1 − v n−1 )
=Xan−1| =X(1 − v n−1 ) =Xv n−1
2 3 Xan−2| X X(1 − v n−2 ) Xv n−2
.. .. .. .. .. ..
. . . . . .
k−1 k Xan−(k−1)| X X(1 − v n−(k−1) ) Xv n−(k−1)
.. .. .. .. .. ..
. . . . . .
n−1 n Xa1| X X(1 − v) Xv
n n+1 0

capital insoluto al principio del periodo n

= (capital insoluto al principio del periodo n − 1) − (capital contenido en el pago n − 1)

5.2. Amortización con pagos iguales


Si va a pagarse la deuda mediante n pagos anuales iguales a X, el pago X satisface:

C = Xan|

C es la deuda original X es el pago anual n es el número de pagos


por lo tanto
C
X= .
an|

Y la tabla de amortización, que debe incluir el capital insoluto al principio del periodo, el
pago que se va a realizar, los intereses contenidos en el pago y el capital contenido en el pago se
va a calcular como:
Las columnas Tiempo y Periodo son columnas alternativas. La columna Periodo también
indica el número de pago que se realiza.

Ejemplo 5.1 Una deuda de $20,000.00 va a ser amortizada mediante 5 pagos iguales vencidos.
Si la tasa de interés efectiva anual es del 4 %, encontrar el pago anual y construir la tabla de
amortización correspondiente.

Xan|

20, 000
X= 1−(1.04)−5
= 4, 492.54.
0.04
Matemáticas Financieras. Mercedes Gregorio 91

Tiempo Periodo Capital insoluto al Interés contenido Capital contenido


[t, t + 1) principio del periodo en el pago en el pago
(m) (m) (m)
0 1 C = Xa Xa i m X X n
m − m (1 − v )
n| n|
=X n
m (1 − v ) =X
mv
n
(m) (m) (m) 1
1 X n
m 2 Xa − mv Xa 1 i m X − X(1 − v n− m )
n| n− m |
(m) 1 1
=Xa 1
= m (1 − v n− m )
X
=X
mv
n− m
n− m |
(m) 2 2
2 X
m 3 Xa 2 m (1 − v n− m ) X n− m
mv
n− m |
.. .. .. .. ..
. . . . .
(m) X n−1 ) X n−1
1 m+1 Xa m (1 − v mv
n−1|
.. .. .. .. ..
. . . . .
k−1 (m) X n− k−1 X n− k−1
m k Xa m (1 − v
m )
mv
m
n− k−1
m
|
.. .. .. .. ..
. . . . .
(m) 1 1
1 X X m
n− m nm Xa 1 m (1 − v )
m
mv
m
|
n nm + 1 0

Periodo Capital insoluto al Interés contenido Capital contenido


principio del periodo en el pago en el pago
1 20000 20000(0.04)=800 4492.54-800=3692.54
2 20000-3692.54=16307.46 16307.46(0.04)=652.30 4492.54-652.30=3840.24
3 16307.46-3840.24=12467.22 12467.22(0.04)=498.69 4492.54-498.69=3993.85
4 12467.22-3993.85=8473.37 8473.37(0.04)=338.93 4492.54-338.93=4153.61
5 8473.37-4153.61=4319.76 4317.76(0.04)=172.79 4492.54-172.79=4319.75
6 4319.76-4319.75=0.01
2462.71 19999.99

También podemos considerar la amortización con pagos iguales con pagos hechos de forma
más frecuentemente que anual.
Si va a pagarse la deuda mediante nm pagos anuales iguales a X X
m , el pago m satisface:

(m)
C = Xa
n|

X
C es la deuda original m es el pago cada m-ésimo de año nm es el número de pagos
por lo tanto

X C
= (m)
.
m ma
n|

Y la tabla de amortización, que debe incluir el capital insoluto al principio del periodo, el
pago que se va a realizar, los intereses contenidos en el pago y el capital contenido en el pago se
va a calcular como:
92 Capítulo 5. Amortización

Ejemplo 5.2 Considérese un préstamo de $1000.00 reembolsable mediante pagos semestrales


durante un periodo de 3 años al 4 % de interés anual convertible semestralmente.
Calcular el pago semestral y construir la tabla de amortización.
(2)
Xa = 1000
3|

1
X (1000) 500 500
= 2 1−v3 = i(2)
= 1−(1.02)−6
= 178.53
2 2
1−(1+ 2 )−6
i 2 0.04
i

Periodo Capital insoluto al Interés contenido Capital contenido


principio del periodo en el pago en el pago
1 1000 1000( 0.04
2 )=20 178.53-20=158.53
2 1000-158.53=841.47 841.47(0.02)=16.83 178.53-16.83=161.70
3 841.47-161.70=679.77 679.77(0.02)=13.60 178.53-13.60=164.93
4 679.77-164.93=514.84 514.84(0.02)=10.30 178.53-10.30=168.23
5 514.84-168.23=346.61 346.61(0.02)=6.93 178.53-6.93=171.60
6 346.61-171.60=175.01 175.01(0.02)=3.50 178.53-3.50=175.03
7 175.01-175.03=-0.02
71.16 1000.02
¿Cuántos intereses están contenidos en el pago 4?
 Ã 
!2 −(3− 4−1
2 )
X³ ´
 i2

1 − v n−( m ) = 178.53 1 −  1 +
k−1
 −3
 = 178.53(1 − (1.02) ) = 10.30
m 2

¿Cuánto capital está contenido en el pago 5?


Ã !2 −(3− 5−1
2 )
X n−( k−1 ) i2
v m = 178.53  1 +  = 178.53(1.02)−2 = 171.60
m 2

¿Cuál es el capital insoluto inmediatamente después del 2o. pago?


à !
(m) (2) (2) 1 − (1.02)−4
Xa = 2(178.53)a = 2(178.53)a = 357.06 = 679.79
k
n− m | 3− 22 | 2| 0.04

5.3. Amortización con un pago irregular


En ocasiones se tiene una deuda que quiere cubrirse en varios pagos de cierta cuantía fijado
de manera autoritaria; en estas condiciones normalmente ocurre que para cubrir el adeudo, debe
hacerse un pago irregular inferior al que se venía haciendo después de un periodo completo.
Para poder resolver estos problemas es necesario primero determinar el número de pagos
iguales que deben hacerse y el monto del pago irregular.

Ejemplo 5.3 Una deuda de $2000 devengando intereses al 4 % efectivo anual debe ser pagada
mediante pagos anuales de $475 más un pago inferior, en caso de ser necesario. Construye la
tabla de amortización.
Matemáticas Financieras. Mercedes Gregorio 93

2000 = 475an|

2000 1 − vn 1 − (1.04)−n
an| = = =
475 i 0.04

ln (1 − 0.04( 2000
475 ))
n= = 4.7
− ln(1.04)
Por lo tanto deben hacerse 4 pagos anuales de $475 y uno menor a los 5 años:

2000 = 475a4| + Xv 5

−4
2000 − 475a4| 2000 − 475( 1−1.04 )
0.04
X= = = 335.55
v5 1.04−5
La tabla de amortización:
Periodo Capital insoluto al Pago Interés contenido Capital contenido
principio del periodo en el pago en el pago
1 2000 475 2000(0.04)=80 475-80=395
2 2000-395=1605 475 1605(0.04)=64.20 475-64.20=410.80
3 1605-410.80=1194.20 475 1194.20(0.04)=47.77 475-47.77=427.23
4 1194.20-427.23=766.97 475 766.97(0.04)=30.68 475-30.68=444.32
5 766.97-444.32=322.65 335.55 322.65(0.04)=12.91 335.55-12.91=322.64
6 0.01

5.4. Fondos de amortización


En el método de fondo de amortización para liquidar una deuda el acreedor recibe pagos
periódicos de intereses y el valor nominal de la deuda al final del plazo (no se reduce la deuda
con los pagos).
Con el objeto de poder hacer el último pago, el deudor crea un fondo por separado en el cual
hace depósitos periódicos iguales de tal forma que después del último depósito el fondo tiene el
valor de la deuda original. Es de suponer que el fondo gana intereses; pero no necesariamente a
la misma tasa que carga el acreedor.
La suma del depósito que se hace en el fondo y los intereses que se pagan en cada periodo se
conoce como el costo de la deuda por periodo.
Si consideramos que el fondo de amortización se va a constituir durante n años con m depósi-
tos cada año a una tasa de interés efectiva anual j y que se pagan intereses por periodo a una tasa
de interés efectiva anual i, el depósito que se hace en cada periodo al fondo, Xm , si consideramos
(m)
depósitos iguales en cada periodo, debe satisfacer que valor nominal de la deuda = Xs ; mien-
n|j ³ ´
(m)
tras los intereses pagados en cada periodo serán iguales a (valor nominal de la deuda) i m .
Donde j (m) e i(m) son las tasas de interés nominales equivalentes a las tasas de interés efectivas
anuales j e i, respectivamente. Por lo tanto
94 Capítulo 5. Amortización

à !
valor nominal de la deuda i(m)
costo de la deuda por periodo = (m)
+(valor nominal de la deuda)
ms m
n|j

 
à !
valor nominal de la deuda  1 (m)  valor nominal de la deuda j (m)
=  (m) + i  = + i(m) .
m s m (1 + j)n − 1
n|j

Si consideramos que i = j, entonces


µ ¶
(valor nominal de la deuda)i(m) (1 + i)n valor nominal de la deuda
costo de la deuda por periodo = =
m (1 + i)n − 1 ma
(m)
n|

el cual es el pago por periodo que se realiza en el pago de una deuda mediante el método de
tabla de amortización.

Ejemplo 5.4 Una deuda de $5000 que devenga intereses al 5 % de interés efectivo anual va a
ser liquidada dentro de 4 años.
Los intereses se pagan anualmente.
El valor nominal de la deuda se va a acumular mediante un fondo de amortización al cual se
le van a hacer aportaciones anuales y otorga un interés del 4 % efectivo anual.

¿Cuál es el importe de cada depósito?


¿Cuál es el costo anual de la deuda?
Construya la tabla del fondo de amortización.

5000 = Xs4|4 %

5000
X= = 1177.45
s4|4 %

Se deben hacer 4 depósitos anuales de 1177.45.


Costo anual de la deuda= pago de intereses + depósito

intereses = 5000i = 5000(0.05) = 250

Costo anual de la deuda=250+1177.45=1427.45


La tabla del fondo de amortización
Periodo Fondo al Intereses Depósito Incremento
principio del periodo del fondo al fondo
1 0 0 1177.45 1177.45
2 1177.45 1177.45(0.04)=47.10 1177.45 47.10+1177.45=1224.55
3 1177.45+1224.55=2402 2402(0.04)=96.08 1177.45 96.08+1177.45=1273.53
4 2402+1273.53=3675.53 3675.53(0.04)=147.02 1177.45 147.02+1177.45=1324.47
5 3675.53+1324.47=5000
Matemáticas Financieras. Mercedes Gregorio 95

En general si la deuda se escribe como deuda = Xsn| la tabla del fondo de amortización se
desarrolla como:
Periodo Fondo al principio Intereses Depósito Incremento
del periodo del fondo al fondo
1 0 0 X X
2 X Xs1| i X X((1 + i) − 1) + X
= Xs1| X((1 + i) − 1) =X(1 + i)
3 Xs2| X((1 + i)2 − 1) X X(1 + i)2
... ... ... ... ...
k Xsk−1| X((1 + i)k−1 − 1) X X(1 + i)k−1
... ... ... ... ...
n Xsn−1| X((1 + i) n−1 − 1) X X(1 + i)n−1
n+1 Xsn|
Por lo tanto,

1. El fondo al principio del k-ésimo periodo es Xsk−1| .

2. En el k-ésimo periodo los intereses que genera el fondo son X((1 + i)k−1 − 1).

3. Al final del k-ésimo periodo el fondo se incrementa en X(1 + i)k−1 .

5.5. Fondo de amortización con pagos variables


Se desea conocer el importe de una deuda que es liquidada por una sucesión de pagos anuales
u1 , u2 , . . . , un hechos a un fondo; estos pagos incluyen los intereses que son pagados anualmente
a una tasa de i % anual. Si el fondo genera intereses a una tasa j % efectivo anual, entonces la
tabla del fondo de amortización se ve de la siguiente forma:
Periodo Fondo al principio Intereses Depósito Incremento
del periodo del fondo al fondo
1 0 0 u1 − Xi u1 − Xi
2 u1 − Xi (u1 − Xi)j u2 − Xi (u1 − Xi)j + (u2 − Xi)
3 (u1 − Xi)(1 + j) (u1 − Xi)(1 + j)j u3 − Xi (u1 − Xi)(1 + j)j
+(u2 − Xi) +(u2 − Xi)j +(u2 − Xi)j + u3 − Xi
4 (u1 − Xi)(1 + j)2 (u1 − Xi)(1 + j) j u4 − Xi (u1 − Xi)(1 + j)2 j
2

+(u2 − Xi)(1 + j) +(u2 − Xi)(1 + j)j +(u2 − Xi)(1 + j)j


+(u3 − Xi) +(u3 − Xi)j +(u3 − Xi)j + u4 − Xi
.. .. .. .. ..
. . . . .
Pn
n+1 k=1 (uk − Xi)(1 + j)n−k

Ejemplo 5.5 Se paga una deuda en 3 años mediante pagos anuales vencidos de $1000, $2000 y
$3000 mediante el método del fondo de amortización. Si la deuda genera intereses al 6 % efectivo
anual y el fondo acumula al 4 % efectivo anual. ¿Cuál es el monto de la deuda?
Sea X el monto de la deuda.
96 Capítulo 5. Amortización

Periodo Fondo al principio Intereses Depósito Incremento


del periodo del fondo al fondo
1 0 0 1000-X(0.06) 1000-X(0.06)
2 1000-X(0.06) (1000-X(0.06))0.04 2000-X(0.06) 2040-X(0.0624)
=40-X(0.0024)
3 3040-X(0.1224) (3040-X(0.1224))0.04 3000-X(0.06) 3121.60-X(0.0649)
=121.60-X(0.0049)
4 6161.60-X(0.1873)
6161.60
Entonces, 6161.60-X(0.1873)=X; por lo tanto X = 1.1873 =5189.59.
Ejemplo 5.6 Resuelve otra vez el problema suponiendo que la deuda genera intereses al 4 %.
Periodo Fondo al principio Intereses Depósito Incremento
del periodo del fondo al fondo
1 0 0 1000-X(0.04) 1000-X(0.04)
2 1000-X(0.04) (1000-X(0.04))0.04 2000-X(0.04) 2040-X(0.0416)
=40-X(0.0016)
3 3040-X(0.0816) (3040-X(0.0816))0.04 3000-X(0.04) 3121.60-X(0.0433)
=121.60-X(0.0033)
4 6161.60-X(0.1249)
6161.60
Entonces, 6161.60-X(0.1249)=X; por lo tanto X = 1.1249 =5477.64.
Si calculamos el valor presente de los pagos:
µ ¶ µ ¶ µ ¶
1 1 2 1 3
1000 + 2000 + 3000 = 5477.64
1.04 1.04 1.04
Del ejemplo se observa que si las tasas de interés del fondo y de la deuda son iguales entonces
los pagos hechos bajo en método de fondo de amortización son los mismos que los hechos bajo
el método de amortización:
n
X
X= (uk − Xi)(1 + j)n−k
k=1
Si i = j
n
X n
X n
X
X= (uk − Xi)(1 + j)n−k = uk (1 + j)n−k − Xj (1 + j)n−k
k=1 k=1 k=1
Pn n−k Pn
k=1 uk (1 + j) k=1 uk (1 + j)n−k
⇒X= Pn =
(1 + j k=1 (1 + j)n−k ) (1 + jsn| )
Pn Pn n
n−k + j)n−k X
k=1 u³k (1 + j) ´ k=1 uk (1
= = = uk (1 + j)−k
n
(1 + j (1+j)j −1 ) (1 + j)n k=1

Ejemplo 5.7 Desarrolla la tabla de amortización correspondiente al ejemplo anterior:


Periodo Capital insoluto al Pago Interés contenido Capital contenido
principio del periodo en el pago en el pago
1 5477.64 1000 5477.64(0.04)=219.11 1000-219.11=780.89
2 5477.64-780.89=4696.75 2000 4696.75(0.04)=187.87 2000-187.87=1812.13
3 4696.75-1812.13=2884.62 3000 2884.62(0.04)=115.38 3000-115.38=2884.62
4 0
Matemáticas Financieras. Mercedes Gregorio 97

5.6. Amortizaciones con cambio de tasas


Pueden considerarse problemas donde durante la vida del crédito la tasa de interés cambie.
Pueden considerarse 2 situaciones donde se de el cambio en tasa de interés:

1. El cambio en tasas de interés se conoce de antemano, por lo que el pago debe contemplar
de antemano el cambio en las tasas.

2. El cambio en tasas de interés no se conoce de antemano, ocurre en el transcurso del tiempo.


En el momento en que se conoce el cambio de tasas se debe reestructurar la deuda.

Supongamos que solamente existe un cambio de tasas a lo largo de la vida del crédito; si existe
un número mayor de cambios la generalización es inmediata. Entonces las tablas de amortización
para cada uno de los casos serían las que a continuación se desarrollan.

5.6.1. Amortizaciones con cambio de tasas conocido de antemano


Si el cambio de tasas es conocido al momento que se firma el contrato, entonces el pago
anual debe considerar ese cambio de tasas. Supongamos que se va a tener una tasa efectiva anual
i % durante los primeros k periodos y una tasa efectiva anual j % durante los siguientes n − j
periodos, entonces el pago anual, X, debe satisfacer que

Xak|i % + Xvik% an−k|j % = C,

lo cual implica que

C
X= .
ak|i % + vik% an−k|j %

Una vez determinado el pago anual se puede observar que la tabla de amortización se cons-
tuiría de la siguiente forma:
Periodo Tasa Cap.ins. inicio Interés contenido Capital contenido
[t, t + 1) del periodo en el pago en el pago
1 i% C = X(ak|i % X(ak|i % + vik% an−k|j % )i X − X(ak|i % + vik% an−k|j % )
+vik% an−k|j % ) = X(1 − vik% + ivik% an−k|j % ) = X(vik% − ivik% an−k|j % )
2 i% X(ak−1|i % X(ak−1|i % + vik−1% an−k|j % )i X − X(ak−1|i % + vik−1% an−k|j % )
+vik−1
% an−k|j % ) = X(1 − vik−1
% + ivik−1
% an−k|j % ) = X(vik−1
% − iv k−1
i % n−k|j % )
a
.. .. .. .. ..
. . . . .
k i% X(a1|i % X(a1|i % + vi % an−k|j % )i X − X(a1|i % + vi % an−k|j % )
+vi % an−k|j % ) = X(1 − vi % + ivi % an−k|j % ) = X(vi % − ivi % an−k|j % )
k+1 j% Xan−k|j % Xan−k|j % j X − X(1 − vjn−k
% )
= X(1 − vjn−k% ) = Xvjn−k
%
n−k−1
k+2 j% Xan−k−1|j % X(1 − vj % ) Xvjn−k−1
%
.. .. .. .. ..
. . . . .
n j% Xa1|j % X(1 − vj % ) Xvj %
n+1 0
98 Capítulo 5. Amortización

Ejemplo 5.8 Una deuda de $10000 se va a pagar mediante 3 pagos vencidos anuales iguales, el
primer año la tasa es de 5 %, el segundo y tercer año es de 3 %. Utiliza el método de amortización.
Calcula en pago anual y realiza la tabla de amortización. (El cambio de tasas se conoce de
antemano)

Xv5 % + Xv5 % v3 % + Xv5 % v32 % = 10000

10000
X= = 3603.95
(1.05)−1 + (1.05)−1 (1.03)−1 + (1.05)−1 (1.03)−2

Periodo Capital insoluto al Pago Interés contenido Capital contenido


principio del periodo en el pago en el pago
1 10000 3603.95 10000(0.05)=500 3603.95-500= 3103.95
2 10000-3103.95=6896.05 3603.95 6896.05(0.03)=206.88 3603.95-206.88=3397.07
3 6896.05-3397.07=3498.98 3603.95 3498.98(0.03)=104.97 3603.95-104.97=3498.98
4 0

5.6.2. Amortizaciones cuando el cambio de tasas no se conoce de antemano


Si el cambio de tasas no se conoce de antemano se calcula el pago anual sin considerar cambio
de tasas, en el momento en que la tasa de interés cambia se recalcula el pago anual. Así el pago
inicial debe satisfacer
C
X1 an|i % = C ⇒ X1 = ,
an|i %

y al momento del cambio de tasa, después de k periodos, el nuevo pago debe satisfacer

X1 an−k|i %
X2 an−k|j % = X1 an−k|i % ⇒ X2 = .
an−k|j %

Una vez determinado el pago anual se puede observar que la tabla de amortización se cons-
tuiría de la siguiente forma:
Tiempo Periodo Tasa Capital insoluto al Pago Interés contenido Capital contenido
t [t, t + 1) principio del periodo en el pago en el pago
0 1 i% C = X1 an|i % X1 X1 (an|i % )i X1 − X1 (1 − vin% )
= X1 (1 − vin% ) = X1 vin%
1 2 i% X1 an−1|i % X1 X1 (1 − vin−1
% ) X1 vin−1
%
.. .. .. .. .. .. ..
. . . . . . .
k−1 k i% X1 an−k+1|i % X1 X1 (1 − vin−k+1
% ) X1 vin−k+1
%
k k+1 j% X2 an−k|j % X2 X2 (1 − vjn−k
% ) X2 vjn−k
%
k+1 k+2 j% X2 an−k−1|j % X2 X2 (1 − vjn−k−1
% ) X2 vjn−k−1
%
.. .. .. .. .. .. ..
. . . . . . .
n−1 n j% X2 a1|j % X2 X2 (1 − vj % ) X2 vj %
n n+1 0
Matemáticas Financieras. Mercedes Gregorio 99

Ejemplo 5.9 Repite el ejemplo anterior si no se conoce de antemano el cambio de tasas.


Al principio el pago anual se calcula como:

10000 = Xa3|5 %

10000
X= 1−1.05−3
= 3672.09
0.05

Al final del primer año se conoce del cambio de tasas. El capital insoluto al final del primer
año es
à !
1 − 1.05−2
3672.09a2|5 % = 3672.09 = 6827.92.
0.05

Tomando en cuenta ese capital insoluto se rediseña el pago:

6827.92 = Xa2|3 %

6827.92
X= 1−1.03−2
= 3568.34
0.03

La tabla de amortización
Periodo Capital insoluto al Pago Interés contenido Capital contenido
principio del periodo en el pago en el pago
1 10000 3672.09 10000(0.05)=500 3672.09-500=3172.09
2 10000-3172.09=6827.91 3568.34 6827.91(0.03)=204.84 3568.34-204.84=3363.5
3 6827.91-3363.50=3464.41 3568.34 3464.41(0.03)=103.94 3568.34-103.94=3464.40
4 0.01

You might also like